#15 Rosh Review

Ace your homework & exams now with Quizwiz!

Question: What are other causes of acquired sideroblastic anemia aside from lead toxicity?

Answer: Pyridoxine deficiency, drug toxicity (chloramphenicol, isoniazid), carcinoma, leukemia, chronic alcoholism and infection. Rapid Review Iron Deficiency Anemia Microcytic Hypochromic Low MCV Low MCHC Low reticulocyte count Low iron High total iron binding capacity Iron supplementation

Question: Name two common viral causes of prenatal hearing loss?

Answer: Rubella and cytomegalovirus. Rapid Review Labyrinthitis Patient with a history of recent upper respiratory infection Complaining of sudden and persistent onset of vertigo and hearing loss lasting several days to a week

Question: What is the first line medication for the treatment of scabies?

Answer: Permethrin 5% cream. Rapid Review Scabies Patient will be complaining of severe pruritus that is worse at night PE will show small papules, vesicles and burrows in the webbed spaces of the fingers and toes Diagnosis is made by microscopic visualization Most commonly caused by Sarcoptes scabiei hominis Treatment is permethrin 5%

Question: Which antiepileptic medication can cause hirsutism and gingival hyperplasia?

Answer: Phenytoin. Rapid Review Febrile Seizure - Simple Patient will be a child 6 mos - 5 yrs Complaining of a single generalized seizure lasting < 15 mins Most commonly caused by rapid rise in temperature Treatment is supportive care

Question: What drug is safe to administer to a patient with wide complex irregular tachycardia?

Answer: Procainamide. Rapid Review Ventricular Tachycardia >3 consecutive ectopic ventricular beats Monomorphic, polymorphic Bidirectional: digoxin toxicity Wide complexes Pulseless: immediate defibrillation Unstable: synchronized cardioversion Stable: procainamide, amiodarone, synchronized cardioversion (refractory) If unsure, manage all wide complex tachycardias as ventricular tachycardia

Question: What are the four types of multiple sclerosis?

Answer: Relapsing remitting, secondary progressive, primary progressive, or progressive relapsing. Rapid Review Multiple Sclerosis Demyelinating disorder Caucasian females Optic neuritis (afferent pupillary defect, pain with eye movement, monocular vision loss, pale optic disc) Internuclear opthalmoplegia Lhermitte phenomenon: spinal electric shock sensation with neck flexion CSF: ↑ IgG protein, WBC pleocytosis Rx: methylprednisolone

Question: What is the most common source of infection in the septic patient?

Answer: Respiratory system. Rapid Review Sepsis Syndromes SIRS: 2 or more of the following T >38° or <36° HR > 90 bpm RR > 20 bpm or PCO2 < 32 mmHg WBC > 12,000 or < 4000 Sepsis: SIRS + infection Severe sepsis: sepsis + organ dysfunction Septic shock: sepsis + refractory hypotension

Question: How frequently do patients with testicular torsion report a history of similar pain?

Answer: Up to 29% of patients with testicular torsion will report similar pain in the past. Rapid Review Testicular Torsion: Patient will be a young male Complaining of intense scrotal pain PE will show exquisite tenderness of the testicle and no cremasteric reflex Diagnosis is made by ultrasound with Doppler Treatment is manual detorsion or surgical

Question: What effect does nitroglycerin have?

Answer: Veno- and arterialdilation, decreasing preload and afterload of the heart. Rapid Review Heart Failure Staging/Classification American Heart Association/American College of Cardiology staging: Stage A: high risk without symptoms/disease Stage B: structural disease without sx Stage C: structural disease + sx Stage D: refractory heart failure New York Heart Association Classification: I: asymptomatic II: sx with ordinary activity III: asymptomatic only at rest IV: sx at rest

Question: Which calcium channel blocker has the greatest effect on the AV node?

Answer: Verapamil Rapid Review Calcium Channel Blocker Toxicity SA/AV node slowing, vasodilation Bradycardia, hypotension, hyperglycemia

Question: Which organism is responsible for producing rice water stools?

Answer: Vibrio cholera. Rapid Review Traveler's Diarrhea Patient with a history of recent travel Complaining of abrupt onset of watery diarrhea, nausea, and abdominal cramping Most commonly caused by enterotoxigenic Escherichia coli (ETEC) Treatment is rehydration and ciprofloxacin

Question: What vitamin does isoniazid deplete?

Answer: Vitamin B6 (pyridoxine). Rapid Review Hypervitaminosis A: Patient with a history of intake of cod liver oil or multivitamins Complaining of headaches, facial acne, nausea, joint pain, itchy and peeling skin, dizziness PE will show enlargement of liver and spleen

Question: What age group is most likely to be affected by an initial flare of dyshidrosis?

Answer: Young adults in their 3rd decade of life are most commonly affected initially. Rapid Review Dyshidrotic Eczema Patient will be complaining of intense pruritus on their palms and sides of the fingers PE will show vesicles that appear to contain "grains of tapioca" Treatment is avoidance of long exposure of the hands to water, topical corticosteroids for acute flares

Question: Which type of thyroid cancer is most closely associated with a history of childhood radiation exposure?

Answer: Papillary carcinoma. Rapid Review Thyroid Carcinoma Radiation hx Papillary: MC Medullary: calcitonin Follicular

Question: Which virus is the most common cause of croup?

Answer: Parainfluenza virus. Rapid Review Laryngotracheitis (Croup) Patient will be a non-toxic appearing child, 6 months to 3 years old Complaining of URI symptoms with barky, seal-like cough, inspiratory stridor, low-grade fever Labs will show steeple sign on PA view Most commonly caused by Parainfluenza virus Treatment is steroids, aerosolized epinephrine

Question: Who should receive leukodepleted blood components?

Answer: Patients who are likely to receive multiple platelet or packed red blood cell transfusions in the future (e.g. sickle cell patients).

Question: Which medications are commonly used to prevent contrast-induced allergic reaction and anaphylaxis?

Answer: Oral prednisone plus oral or intravenous diphenhydramine, given prior to a medical procedure in which iodinated medium may be purposely or inadvertently placed intravenously Rapid Review Acute Tubular Necrosis MCC of intrinsic renal failure Types: ischemic and nephrotoxic Nephrotoxins: aminoglycosides > contrast agents Granular "muddy brown" casts

Question: What is the treatment of hemodynamically unstable ventricular tachycardia?

Answer: Electrical cardioversion. Rapid Review Ventricular Tachycardia >3 consecutive ectopic ventricular beats Monomorphic, polymorphic Bidirectional: digoxin toxicity Wide complexes Pulseless: immediate defibrillation Unstable: synchronized cardioversion Stable: procainamide, amiodarone, synchronized cardioversion (refractory) If unsure, manage all wide complex tachycardias as ventricular tachycardia

Question: Which test should all patients with sarcoidosis have annually?

Answer: Electrocardiogram. 5-25% of sarcoid patients will have conduction block and ventricular tachycardia. Rapid Review Sarcoidosis African-Americans, females Primary target organ: lungs Parotid enlargement Hypercalcemia CXR: bilateral hilar adenopathy Biopsy: noncaseating granulomas Steroids

Question: When are patients contagious with measles?

Answer: From 1 to 2 days before symptoms and 4 days after the rash appears. Rapid Review Rubeola (Measles) Fever 3 c's: cough, conjunctivitis, coryza Koplik's spots: red spots with blue/white center (pathognomonic) Rash spreads head to feet

Question: Based on the 2007 Youth Risk Behavior Survey, what percentage of students in grades 9 through 12 reported that they had seriously considered attempting suicide in the 12 months preceding the survey?

Answer: 14.5%. Rapid Review Suicide Protective factors: marriage, pregnancy RFs (SAD PERSONS): Sex (male), Age (teenager or elderly), Depression, Previous attempt, Ethanol/drug use, Rational thinking loss, Sickness (medical illness, 3 or more prescription medications), Organized plan, No Social support, Stated future attempt Most completed suicides involve firearms Most attempted suicides involve ingestions (MC: antidepressants) All overdose patients: obtain acetaminophen level

Question: What type of hearing loss results in patients with a cholesteatoma?

Answer: A cholesteatoma can cause a conductive hearing loss as it obstructs the auditory canal. Rapid Review Acquired Cholesteatoma Patient will have a history of chronic ear infections or tympanostomy tubes Complaining of painless otorrhea PE will show yellow or white mass behind the tympanic membrane Treatment is tympanomastoid surgery

Question: What is the significance of a high amylase level in a pleural fluid sample?

Answer: A high amylase level usually indicates the presence of pancreatitis, esophageal rupture or malignancy. Rapid Review Pleural Effusion Transudate: CHF (most common) Exudate: infection > malignancy, PE ↓ Breath sounds + dull percussion + ↓ tactile fremitus CXR: blunting of the costophrenic angle

Question: What test can help to determine if a patient is surreptitiously self-administering insulin?

Answer: A low C peptide level with a high insulin level is diagnostic for factitious hypoglycemia. Rapid Review Hypoglycemia Usually glucose <60 Confusion, agitation, unresponsiveness Tachycardia, diaphoresis, tremulousness Focal neurologic deficit Dextrose, thiamine, glucagon

Question: What neuromuscular receptors are involved in cases of myasthenia gravis?

Answer: Acetylcholine nicotinic postsynaptic receptors. Rapid Review Myasthenia Gravis Patient will be complaining of proximal muscle weakness, ptosis, and diplopia that is worse at the end of the day PE will show ice test improves sx Diagnosis is made by edrophonium (tensilon) test, EMG Most commonly caused by autoimmune destruction of acetylcholine receptors Treatment is acetylcholinesterase inhibitors, such as pyridostigmine Comments: associated with thymoma

Question: When should treatment be initiated in pregnancy to prevent neonatal infection?

Answer: Around 32 weeks gestation. Rapid Review Condyloma Acuminata Patient will be complaining of genital lesions PE will show cauliflower-like lesion Most commonly caused by HPV 6 & 11 Comments: most common STD

Question: What is the function of the cochlea's round window?

Answer: As sound causes the stapes and oval window to move in, the round window and its membrane move out. If absent, the cochlear fluid would be incompressible, resulting in hearing loss. Rapid Review Hearing Loss Conductive Weber: localizes to affected ear Rinne: abnormal (BC > AC) Most commonly caused by otitis media, serous otitis, and cerumen impaction Sensorineural Weber: localizes to unaffected ear Rinne: normal (AC > BC) Most commonly caused by excessive noise exposure, drugs, and normal aging

Question: Why is phimosis less common as patients age?

Answer: At birth, only 4% of boys have a fully retractable foreskin. Physiologic changes increase this rate until 4 years of age when 90% are fully retractable. Rapid Review Phimosis Inability to retract foreskin behind glans Rx: hygeine, topical steroids Rx if vascular compromise: dorsal slit Definitive rx: circumcision

Question: What is the long-term treatment for paroxysmal supraventricular tachycardia (PSVT)?

Answer: Calcium channel blockers, beta-blockers and radiofrequency catheter ablation. Rapid Review Supraventricular Tachycardia All tachydysrhythmias that arise above the bifurcation of the bundle of His Characteristics: Atrial rate 120-200 Rhythm: regular Narrow QRS Causes Pre-excitation syndromes (WPW) Mitral disease Digitalis toxicity Drugs and toxins Hyperthyroidism Treatment: Vagal maneuvers (Valsalva) Adenosine (first-line medication), ßBs, CCBs Unstable: synchronized cardioversion

Question: Which vagal maneuver is relatively contraindicated in the elderly population?

Answer: Carotid massage, as the risk of embolic stroke from underlying carotid atherosclerosis is present. Rapid Review Supraventricular Tachycardia All tachydysrhythmias that arise above the bifurcation of the bundle of His Characteristics: Atrial rate 120-200 Rhythm: regular Narrow QRS Causes Pre-excitation syndromes (WPW) Mitral disease Digitalis toxicity Drugs and toxins Hyperthyroidism Treatment: Vagal maneuvers (Valsalva) Adenosine (first-line medication), ßBs, CCBs Unstable: synchronized cardioversion

Question: What are the most common cancers associated with small bowel obstruction?

Answer: Colonic and ovarian malignancies. Rapid Review Small Bowel Obstruction Patient with a history of pelvic surgery Complaining of bilious vomiting PE will show high pitched bowel sounds X-ray will show dilated bowel, air fluid levels, "stack of coins" or "string of pearls" sign Diagnosis is made by CT Treatment is NGT, surgery

Question: What complications can arise from viral croup?

Answer: Complications can include dehydration, otitis media, and pneumonia. Rapid Review Laryngotracheitis (Croup) Patient will be a non-toxic appearing child, 6 months to 3 years old Complaining of URI symptoms with barky, seal-like cough, inspiratory stridor, low-grade fever Labs will show steeple sign on PA view Most commonly caused by Parainfluenza virus Treatment is steroids, aerosolized epinephrine

Question: Up to 40% of patients with GCA present with which atypical symptoms?

Answer: Cough, choking sensation, neuropathy and extremity claudication. Rapid Review Temporal Arteritis (Giant Cell Arteritis) Patient will be a woman > 50 y/o Complaining of monocular visual loss, unilateral headache, jaw claudication PE will show tender temporal Artery Labs will show ESR > 50 Diagnosis is made by temporal artery biopsy Treatment is high dose steroids ASAP Comments: Associated polymyalgia rheumatica

Question: What are the nodular deposits seen in gout?

Answer: Deposits of uric acid crystals, or tophi, which can be seen in subcutaneous tissues, tendons, cartilage, and bone. Rapid Review Gout Patient will be a middle-aged man Complaining of acute onset of pain in the first MTP (Podagra) Labs will show needle-shaped crystal with negative birefringence Most commonly caused by uric acid crystals Treatment is: Acute: NSAID's Chronic: allopurinol or colchicine Comments: can be triggered by loop and thiazide diuretics

Question: List some causes of secondary hypertension?

Answer: Diabetes, glomerulonephritis, renal atherosclerosis, Cushing's syndrome, hyperaldosteronism, pheochromocytoma, obstructive sleep apnea, aortic coarctation, meds (OCPs, NSAIDs, steroids, cyclosporine). Rapid Review Hypertension: Eighth Joint National Committee (JNC 8) Recommendations PreHTN: systolic blood pressure (SBP) 120-139 mmHg or diastolic blood pressure (DBP) 80-89 mmHg Stage I HTN: SBP 140-159 mmHg or DBP 90-99 mmHg Stage II HTN: SBP >160 mmHg or DBP >100 mmHg Treatment goals: >60 years: SBP<150, DBP<90 All others: SBP<140, DBP<90 1st line rx for general population: thiazide, CCB, ACEI, or ARB 1st line rx for African Americans: CCB or thiazide Chronic kidney disease: Rx should include ACEI or ARB

Question: What vaccine should be given to all patients on chronic salicylate therapy?

Answer: Inactivated influenza vaccine. Rapid Review Rheumatic Fever Patient with a history of GAS infection Complaining of fever, red skin lesions on the trunk and proximal extremities, and small, non-tender lumps located over the joints PE will show JONES criteria: Joints, Oh, no carditis!, Nodules, Erythema marginatum, Sydenham's chorea Labs will show anti-streptolysin O, anti-DNase B, positive throat culture, or positive rapid antigen test Treatment is antibiotics, NSAIDs Comments: Modified Jones Criteria for a first episode of acute rheumatic fever: need 2 major or 1 major and 2 minor

Question: What is the underlying pathology of idiopathic primary varicocele?

Answer: Incompetent venous veins, analogous to varicose veins. Rapid Review Varicocele Adolescent males ↑ Venous pressure → Dilation of pampiniform venous plexus along the spermatic cord → scrotal enlargement Most common side: left "Bag of worms" Ultrasound

Question: What is the treatment for life-threatening hypermagnesemia?

Answer: Intravenous calcium (either calcium chloride or calcium gluconate) and dialysis.

Question: When does the vessel-expansion, typically seen in the jugular vein as a double-pulsation, occur in the cardiac cycle?

Answer: Just after S1 and during S2. Rapid Review Heart Sounds S1: mitral and tricuspid valve closure S2: aortic and pulmonary valve closure S3: in early diastole During rapid ventricular filling phase Large amount of blood striking a very compliant left ventricle (LV) Normal in children, pregnant women S4 ("atrial kick"): Late diastole Blood flowing against noncompliant LV

Question: What is normal intraocular pressure?

Answer: Less than 20 mm Hg. Rapid Review Acute Angle-Closure Glaucoma Patient will be entering a dark room or movie theater Complaining of acute unilateral painful vision loss, vomiting, and seeing halos around lights PE will show cloudy cornea and fixed mid-dilated pupil Labs will show ↑ IOP ( > 21 mm Hg) Treatment is topical ßBs, carbonic anhydrase inhibitors, steroids, miotics

Question: Which medications used to treat diabetes type 2 has the greatest effect in decreasing hemoglobin-A1c levels?

Answer: Metformin, the glinides and the sulfonylureas. Rapid Review Diabetes Mellitus Type 2 ↑ Insulin resistance Obesity, family hx Candidiasis Blurry vision Hyperosmolar hyperglycemic state Lifestyle changes First line pharmaceutical rx: metformin

Question: What is the treatment for toxic acetaminophen ingestion?

Answer: N-acetylcysteine. Rapid Review Ethylene Glycol Toxicity Patient will be complaining of nausea and vomiting PE will show CNS depression and acute mental status change Labs will show calcium oxalate crystals in the renal tubules, renal tubular necrosis, hypocalcemia Treatment is fomepizole Comments: Commonly found in antifreeze

Question: What is the treatment for Chagas disease?

Answer: Nifurtimox or benznidazole. Rapid Review Myocarditis Leads to dilated cardiomyopathy, CHF Sudden death in young adults Idiopathic > viral (Coxsackie) Viral prodrome Positive troponin ST-segment elevation ECHO shows global hypokinesis Supportive management, possible transplant

Question: Is magnetic resonance angiography recommended in the diagnosis of PE?

Answer: No, its poor resolution caused by cardiopulmonary motion artifact limits its usefulness. Rapid Review Pulmonary Embolism 95% arise from deep leg veins Sudden onset of symptoms in 50% SOB, CP, tachypnea ECG: sinus tachycardia, nonspecific ST-T changes, right heart strain, S1Q3T3 (classic finding) CXR: nonspecific abnormalities, Hampton's hump (pleural-based wedge infarct), Westermark's sign (vascular cut-off sign) V/Q scan: usually nondiagnostic Low clinical suspicion: negative D-dimer excludes PE Dx of choice: CTPA Treatment: Anticoagulation Thrombolytics (if massive and HD unstable or submassive with shock, respiratory failure or evidence of moderate to severe RV strain) Embolectomy (last resort)

Question: Do patients with lymphocutaneous sporotrichosis require admission?

Answer: No; unless there is disseminated disease or involvement of joints, bones, or tendons, it can be treated as an outpatient. Rapid Review Sporotrichosis PE will show red ulcer with lymphcutaneous spread Most commonly caused by rose thorn infection, Sporothrix schenckii Treatment is itraconazole

Question: Which racial/ethnic groups have the highest SUDI mortality rates?

Answer: Non-Hispanic African American and American Indian/Alaska Native infants.

Question: In a patient with localized disease, which type of germ cell tumor is curable by orchiectomy alone?

Answer: Nonseminoma. Rapid Review Testicular Cancer Patient will be a man 20 - 35 years old Complaining of a testicular lump PE will show painless, hard, fixed mass Labs will show increased beta-hCG, alpha-fetoprotein (AFP), or lactate dehydrogenase (LDH) Diagnosis starts with ultrasound Comments: Most common risk factor: cryptorchidism, Most common tumor: seminoma

Question: What blood smear finding is suggestive of microangiopathic hemolytic anemia?

Answer: Schistocytes (helmet cells). Rapid Review Hematuria Trauma: bladder injury, ureter injury, retropertioneal injury Anemia, thrombocytopenia, renal insufficiency: HUS Flank pain: nephrolithiasis Urine + for heme and - for RBCs: rhabdomyolysis Dysuria, frequency: UTI Hearing loss: Alport syndrome Hemoptysis: Goodpasture syndrome Immigrant: Schistosomiasis Nephrotic syndrome, flank pain: renal vein thrombosis Proteinuria, RBC casts: glomerulonephritis Recent URI: glomerulonephritis, IgA nephropathy

Question: What jagged, irregularly shaped, red blood cells indicate disseminated intravascular coagulation, a complication of acute myelogenous leukemia?

Answer: Schistocytes. Rapid Review Acute Myeloid Leukemia (AML) Patient will be an adult Complaining of fever, fatigue, anemia, easy bruising or bleeding, petechiae, bone and joint pain, and persistent or frequent infections PE will show hepatosplenomegaly Labs will show Auer rods, normocytic, normochromic anemia

Question: What is a common side effect of SSRI treatment that patients often complain about?

Answer: Sexual dysfunction.

Question: Which medications are recommended for treating WPW during pregnancy?

Answer: Sotalol or flecainide. Rapid Review Wolff-Parkinson-White (WPW) Syndrome Accessory pathway (bundle of Kent) connects atria to ventricles, bypassing AV node ECG: short PR interval, delta wave, wide QRS Orthodromic reentry tachycardia Reentry circuit traveling down AV node → narrow complexes Rate: controlled by AV node Rx: ACLS Antidromic reentry tachycardia Reentry circuit traveling down bundle of Kent → wide complexes Rate: controlled by bundle of Kent (capable of rapid ventricular response) Rx: procainamide WPW with atrial fibrillation QRS complexes vary in width and amplitude Rate >250 bpm Definitive rx: radiofrequency ablation

Question: What is the treatment of tender breast engorgement in a non-breast feeding postpartum woman?

Answer: Supportive bra, cold cabbage leaves, and ice packs. Rapid Review Mastitis Lactating women 2-3 weeks postpartum Staphylococcus > Streptococcus, H. Influenza Instruct mothers to keep nursing on side of infection First generation cephalosporin or beta-lactamase resistant PCN

Question: What is a complex concussion?

Answer: Symptoms persist >10 days and involve symptom recurrence with exertion and prolonged cognitive impairment. Rapid Review Concussion Brief LOC, amnesia No focal neurologic deficits Negative CT scan

Question: Why is the MMR-Varicella combined vaccine given to 4-year-olds and not 1-year-olds?

Answer: The risk of febrile seizures increases from 1 in 3,000 in 4-year-olds to 1 in 1,250 in 1-year-old children.

Question: What blood pressure defines a hypertensive emergency?

Answer: There is no defining blood pressure; all that is needed is an elevated pressure in the setting of acute end-organ damage. Rapid Review Hypertensive Emergency Hypertension with acute end-organ system injury Encephalopathy, cardiac ischemia, renal ischemia Objectives: reduce MAP 25% in first hour, normalize BP over the next 8 to 24 hours Reduction of MAP > 25% may cause end-organ ischemia IV antihypertensives (labetalol or nicardipine)

Question: What is methadone maintenance therapy?

Answer: This is also a substitute maintenance medication for those dependent on opiates.. Rapid Review Substance Dependence Diagnosis requires ≥3 of the following: Physiologic tolerance or withdrawal Desire/attempts to ↓ use Activities focused on substance Missing important activities for substance Persistent use in spite of awareness of risks

Question: What is the recommend splint for a scaphoid fracture?

Answer: Thumb spica splint. Rapid Review Scaphoid Fracture FOOSH Snuff box tenderness Possibly negative radiographs Complication: avascular necrosis Thumb spica splint

Question: What is the study of choice in an unstable patient to confirm the diagnosis of aortic dissection?

Answer: Transesophageal echocardiogram. Rapid Review Aortic Dissection Patient will be older With a history of HTN, smoking, Marfan syndrome Complaining of sudden "ripping" or "tearing" CP radiating to back PE will show asymmetric pulses/BP CXR will show widened mediastinum Diagnosis is made by CT or transesophageal echocardiogram (TEE) Treatment is reduce BP, surgery

Question: True or false: All patients with chronic obstructive pulmonary disease should be prescribed a short-acting bronchodilator?

Answer: True.

Question: What is the most common cause of infectious primary adrenal insufficiency worldwide?

Answer: Tuberculosis. Rapid Review Primary Adrenal Insufficiency (Addison's Disease) Patient will be complaining of abdominal pain, nausea, vomiting, diarrhea, fever, and confusion PE will show hyperpigmentation of skin and mucus membranes and hypotension Labs will show hyponatremia and hyperkalemia Most commonly caused by autoimmune Treatment is hydrocortisone

In which setting is allopurinol contraindicated for the treatment of gout? Acute attack of gout Alcoholic patient Hyperuricemia greater than 8.0 In combination with colchicine

Correct Answer ( A ) Explanation: Allopurinol therapy should never be initiated until an acute attack has subsided. Allopurinol is a xanthine oxidase inhibitor that decreases uric acid production but also produces a more soluble metabolite. Allopurinol is therefore effective regardless of the cause of the hyperuricemia. Gout is caused by deposition of uric acid crystals in the synovium, bursae, tendon sheaths, skin, heart valves, and kidneys, which can lead to arthritis, tophi, renal stones, and gouty nephropathy. Typically, patients present with acute, exquisitely tender, monoarticular arthritis. The joint is usually warm and erythematous. The first metatarsophalangeal joint (MTP) joint is most commonly affected. Detecting negatively birefringent, needle-like crystals in the synovial fluid from an arthrocentesis confirms the diagnosis. First line treatment are NSAIDs. Indomethacin has traditionally been the NSAID of choice, but any NSAID can be used with similar efficacy. Ethanol metabolism blocks renal excretion of uric acid which increases the risk for developing gouty arthritis. However, allopurinol is not contraindicated in individuals who consume ethanol (B). Hyperuricemia (C) is present (usually >8 mg/dL) in the gout patient, and when symptomatic, these patients should be treated. Colchicine (D) can be used in combination with allopurinol to help decrease the likelihood of a gouty flare while the uric acid level is decreasing after an acute attack has subsided. Colchicine decreases inflammation associated with lactic acid production and phagocytosis of urate; it terminates most gouty attacks within 6 to 12 hours but is limited by GI side effects.

A patient presents with acute dyspnea and pleuritic chest pain. You suspect pulmonary embolism. Which of the following is the most appropriate test to confirm the diagnosis of a pulmonary embolus? Computed tomography angiography D-dimer assay Echocardiography Ventilation-perfusion scan

Correct Answer ( A ) Explanation: Clinical impression alone does not suffice for the diagnosis of acute pulmonary embolism. Diagnosis is difficult due to the low specificity of the common presenting symptoms of acute dyspnea, cough and pleuritic chest pain. Therefore, supplemental testing is used to effectively rule-in or rule-out pulmonary embolism (PE). Routine laboratory tests are nonspecific. Arterial blood gas measurements have a limited role in diagnosing PE. Electrocardiogram and chest radiography abnormalities found in patients with PE are also commonly found in patients without PE, which limits the diagnostic usefulness of these tests. Pulmonary angiography is the definitive, or "gold standard", test for the diagnosis of PE. It is performed by injecting contrast intravenously via the femoral vein. The radiologist looks for a filling defect or abrupt cutoff in the pulmonary vasculature which concretely indicates the presence of an embolus. Helical CT with pulmonary angiography is being used more and more as the diagnostic test for PE as it can detect not only an embolus, but also alternative pulmonary abnormalities which may be causative of the patient's symptoms. However, one must take into account the radiation risk of CT versus the diagnostic benefit or necessity of this test. D-dimer assays (B) for the diagnosis of PE have been extensively studied. They are best characterized as having good sensitivity and negative predictive value, but poor specificity and positive predictive value. Low D-dimer levels appear best at excluding, not including, the diagnosis of PE. Echocardiography (C) reveals PE-suggestive abnormalities in only 30-40% of patients with actual PE. V/Q scanning (D), although counterintuitive, generates data that is insufficient to exclude or confirm the diagnosis of PE.

You are called to the emergency department to see a five-year-old boy for ingestion. He was playing in the garage when his mother found him with an opened bottle of antifreeze. The liquid was all over his mouth and clothes. He had two episodes of vomiting and fell asleep on the way to the hospital. On physical examination, you note tachycardia. Which of the following laboratory abnormalities would you expect in this type of ingestion? Calcium oxalate crystals in urine Decreased lactate levels Hypercalcemia Non-anion gap metabolic acidosis

Correct Answer ( A ) Explanation: Ethylene glycol is commonly found in antifreeze. Unintentional ingestion is the most common exposure in children, and even small-volume ingestions of concentrated products have the potential for severe toxicity. Early symptoms begin with nausea, vomiting, CNS depression. Late manifestations include anion gap metabolic acidosis, hypocalcemia, and kidney failure (secondary to deposition of calcium oxalate crystals in the renal tubules). It can be diagnosed by ethylene glycol blood concentrations. Osmolar gap may also be used as a surrogate marker. Examination of the urine with a Wood lamp is neither sensitive nor specific for ethylene glycol ingestion. Calcium oxalate crystals can be seen on urine microscopy. The evaluation of patients should include electrolytes (including calcium), acid-base status, kidney function, and ECG. Gastric decontamination is generally not of value because ethylene glycol is rapidly absorbed. The classic antidote for ethylene glycol poisoning was ethanol. However ethanol has been replaced by fomepizole, a potent competitive inhibitor of alcohol dehydrogenase. This is because of the ease of administration, lack of CNS and metabolic effects, and overall excellent patient tolerability profile of fomepizole. Decreased lactate levels (B) is wrong because patients with ethylene glycol poisoning can have elevations in their serum lactate concentration from acidosis. Hypercalcemia (C) is not expected because ethylene glycol can cause hypocalcemia. Non-anion gap metabolic acidosis (D) is not correct. On the other hand, there is profound high anion gap metabolic acidosis (serum bicarbonate less than 8 meq/L) in ethylene glycol intoxication.

A two-year-old previously healthy boy is witnessed having a tonic-clonic seizure in the emergency room. Mom originally brought him in due to a fever of 39°C. The episode lasted five minutes, and the patient returned to baseline mental status. Which of the following is the most appropriate next step in management? Administer antipyretics Initiate anticonvulsant therapy Perform a lumbar puncture Perform an EEG

Correct Answer ( A ) Explanation: Febrile seizures are seizures that occur between the ages of six months and 60 months with a temperature of 38°C or higher, that are not the result of a central nervous system infection or any metabolic imbalance and that occur in the absence of a history of prior afebrile seizures. A simple febrile seizure is a primary generalized, usually tonic-clonic attack associated with fever, lasting for a maximum of 15 minutes and not recurring within a 24-hour period. A complex febrile seizure is more prolonged (>15 min), is focal or recurs within 24 hours. Treatment is mainly supportive for simple febrile seizures and involves evaluation of the fever source, control of fevers with antipyretics such as acetaminophen (not aspirin), and reassurance of the parents. An EEG (D) is not indicated unless the seizure is focal or atypical (complex febrile seizure). Moreover, if an EEG is indicated, it is usually delayed two weeks after the initial event. Anticonvulsant therapy (B) is not indicated for children with one or more simple febrile seizures. A lumbar puncture (C) should be strongly considered in children ages six to 12 months with a febrile seizure who have not received or are behind on immunizations, have been exposed to other children with serious bacterial infections, or who have been pretreated with antibiotics. The identification of a source of fever, such as UTI or otitis media, does not eliminate the possibility of meningitis. For children > 18 months of age, a lumbar puncture is only indicated in the presence of clinical signs and symptoms of meningitis or other intracranial infection

A 58-year-old man presents with 3 months of headache and diplopia. He also reports that chewing tough foods has been progressively painful. Laboratory testing reveals a normocytic anemia and ESR of 88 mm/hour. A brain CT is normal. Which of the following is the most appropriate initial intervention? Intravenous methylprednisolone Oral methotrexate Temporal artery biopsy Ultrasonography

Correct Answer ( A ) Explanation: Giant cell arteritis (GCA), as known as temporal arteritis, is the most common primary vasculitis of the elderly, and predominately affects those aged over 50 years. It is a syndrome of systemic inflammation which mainly affects the branches of the internal and external carotid arteries, but it can affect any vessel in the body. The most common symptoms are a temporal or occipital headache, jaw claudication (mandibular, tooth and tongue pain with chewing tough foods), visual changes (partial or complete blindness, diplopia, visual field cuts or amaourosis fugax) and arthralgias. It is associated with constitutional symptoms, anemia, an enlarged tender nodular erythematous temporal artery, ESR > 50 mm/hour and polymyalgia rheumatica. Temporal artery biopsy confirms the diagnosis. Since unilateral partial or complete blindness occurs in up to 20% of patients with GCA, immediate treatment is necessary when this condition is suspected. High-dose corticosteroids are the standard treatment, and typical options include 40-60 mg per day of oral prednisone, or intravenous methylprednisolone. High-dose therapy is usually continued for 2-4 weeks, and titrated against visual and headache improvement, not serial ESR monitoring. After symptoms lessen, steroids are slowly tapered over 6 months, with complete tapering off in 2 to 3 years. As such, the clinician must monitor for complications of long-term corticosteroid therapy. These include hypertension, diabetes, osteoporosis, steroid myopathy (proximal weakness), fluid retention, bruising, insomnia, restlessness, hypomania and hypercholesterolemia. Adjuvant methotrexate (B) is not routinely recommended for treating GCA as studies of its efficacy are inconclusive. If GCA is suspected, intravenous methylprednisolone should be started even before a temporal biopsy (C). Steroids will not affect the validity of a biopsy result until about 3-4 weeks of treatment, however, a delay in steroids may lead to blindness. There is some evidence of the usefulness of temporal artery ultrasonography (D) for detecting the best area of the artery to biopsy. However, a negative ultrasound does not rule out GCA, and furthermore, some studies have shown that temporal artery ultrasound sometimes cannot differentiate degenerative changes from inflammatory changes.

A 3-year-old girl presents to the ED with 1 hour of a barking cough and inspiratory stridor at rest. On exam, she has mild retractions but is not hypoxic. Which of the following interventions has been shown to reduce hospital length of stay in moderate to severe croup? Dexamethasone Heliox Humidified air Racemic epinephrine

Correct Answer ( A ) Explanation: Glucocorticoids are considered the mainstay of treatment for moderate to severe croup. They are the only treatment that alters the natural course of the illness. Dexamethasone has been shown to reduce the use of epinephrine, severity scores at 6 hours, hospital lengths of stay, and readmissions. In some studies, it has also reduced the rate of intubation. Neither heliox (B) nor humidified air (C) has shown any benefit in the treatment of croup. Racemic epinephrine (D) works acutely, likely by causing local vasoconstriction and reducing hyperemia and edema. It has a very transient effect, however.

A 4-year-old boy who has not received routine childhood vaccinations presents with fever, cough, coryza, and conjunctivitis for 3 days. Today his mother noted an erythematous macular rash over his trunk and extremities. Which of the following is the most likely diagnosis? Measles Mumps Roseola Rubella

Correct Answer ( A ) Explanation: Measles is spread by direct contact with infectious droplets or airborne dissemination. From exposure to the onset of symptoms takes 8-12 days; the onset of the rash, about 14 days. Clinically, measles is associated with fever, conjunctivitis, coryza, and cough (the 3 Cs). The rash is a discrete red maculopapular rash that first appears on the forehead, becoming coalescent as it spreads down the trunk to the feet. The rash fades in the same head-to-feet pattern as it appeared. Measles is also associated with Koplik's spots. These are 1- to 3-mm bluish-white spots on a bright red surface that appear first on the buccal mucosa opposite the lower molars. They are a pathognomonic exanthema of measles and appear approximately within 48 hours after the onset of symptoms. Mumps (B) typically affects the salivary glands with swelling and pain. Other areas that may be infected with mumps include the testes, pancreas, and meninges. Roseola (C) is typically seen in children between 6 months and 2 years of age and presents with a history of high fever of 3 days' duration and mild symptoms. The fever abates abruptly, followed by the appearance of a macular rash on the trunk and thighs. It is caused by Human herpesvirus 6 (HHV6). Rubella (D) is associated with a maculopapular rash that first appears on the face and spreads downward to involve the trunk and extremities (similar to measles). The rash on the face fades on Day 2, and the rash on the trunk becomes coalescent. By the Day 3, the rash disappears, which is why rubella is also called 3-day measles.

A 26-year-old woman presents with dizziness and palpitations. She reports episodes of these symptoms beginning about 1 week ago, which initially only lasted a few minutes. However, for the past two days, she has had about 4 episodes a day which last about 20 minutes each. Her social history is significant for heavy caffeine intake. Her pulse is 166 bpm and her blood pressure is 140/70. Her rhythm strip is seen above. Which of the following best describes the most likely underlying cardiac pathology responsible for this patient's symptoms? Conduction reentry Dilated cardiomyopathy Myocardial infarction Valvular abnormality

Correct Answer ( A ) Explanation: Paroxysmal supraventricular tachycardia (PSVT) is an acute onset tachydysrhythmia that can affect any age group. The dysrhythmia is caused by a reentrant conduction pathway, most commonly in the atrium or atrioventricular node. Common triggers include atrial or atrioventricular premature beats, hyperthyroidism, caffeine and drugs. The common symptoms of paroxysmal supraventricular tachycardia include palpitations, dizziness, dyspnea, syncope, angina, fatigue, nausea and sweating. The most common symptoms are palpitations and dizziness. The chest pain which accompanies this class of dysrhythmias is usually due to the tachycardia itself. Hemodynamically stable patients should first attempt vagal maneuvers. If unsuccessful, AV-nodal blocking agents can be administered such as adenosine or calcium channel blockers. If the patient is hemodynamically unstable, she should immediately undergo cardioversion. Dilated cardiomyopathy (B) and valvular abnormalities (D) are more affiliated with atrial fibrillation. Myocardial infarction (C) is more closely associated with ventricular tachycardia or fibrillation.

Which of the following statements is true regarding teenage suicide? Attempts are more common in girls than boys Boys complete suicide most commonly by ingesting a poisonous substance Girls complete suicide more often than boys Suicide rates are decreasing in teenagers

Correct Answer ( A ) Explanation: Suicide attempts are more common in girls than boys (approximately 3:1), whereas boys complete suicide at a rate 4 times that of girls and represent 79.4% of all suicides. Youth suicide is a major and preventable public health problem. It ranks as the 3rd and 4th leading causes of death among young people ages 15-24 year and 10-14 year, respectively. Suicide is very rare before puberty. Rates of completed suicide increase steadily across the teen years and into young adulthood, peaking in the early 20s. The male:female ratio for completed suicide rises with age from 3:1 in young children to approximately 4:1 in 15-24 year olds, and to greater than 6:1 among 20-24 yr olds. It is estimated that for every completed youth suicide, as many as 200 suicide attempts are made. Ingestion of medication is the most common method of attempted suicide. Boys complete suicide at a rate 4 times that of girls (C). Firearms (B) remain the most commonly used method of completing suicide for boys, whereas girls are more likely to complete suicide by poisoning. In the past 60 years, the suicide rate has quadrupled (D) among 15-24-year old men and has doubled for women of the same age.

A careful cardiac examination requires close attention to the heart sounds. The second heart sound, S2, is produced by which of the following structures? Aortic and pulmonic valves Mitral and tricuspid valves Pericardium and chest wall Posterior and anterior cusps of the mitral valve

Correct Answer ( A ) Explanation: The cardiac cycle begins with ventricular contraction, or systole. The pressure generated closes the mitral and tricuspid valves, producing S1, and opens the aortic and pulmonic valves. As the ventricles relax, the previously "pumped-out" blood pushes back on the heart, closing the aortic and pulmonic valves and producing the second heart sound, S2. The ECG's P wave comes before S1, the QRS complex occurs during S1 and the T wave occurs between S1 and S2. S1 is produced by the closure of the mitral and tricuspid valves (B). A friction rub may be appreciated when there is fluid or irritation between the pericardium and the chest wall (C). Functional abnormalities of the mitral valve's posterior and anterior cusp (D) may lead to a stenotic or regurgitant murmur.

Which of the following is the most common complication of this injury? Avascular necrosis Infection Malunion Osteoarthritis

Correct Answer ( A ) Explanation: The radiograph demonstrates a fracture at the waist or middle third of the scaphoid. The blood supply to the scaphoid penetrates the cortex on the dorsal surface near the tubercle waist area (distal aspect of scaphoid). Therefore, there is no direct blood supply to the proximal portion of the bone. Because of this tenuous blood supply, scaphoid fractures have a tendency to develop avascular necrosis (AVN). The more proximal the fracture, the greater the likelihood of developing AVN. Infection (B) is an uncommon complication of an isolated scaphoid fracture. Malunion (C) is a complication due to improper healing of the fracture. It mainly occurs when a scaphoid fracture goes unrecognized or there is early discontinuation of immobilization. This can lead to arthritis (D) over time due to misalignment from the abnormal motion and collapse of the bone fragments. If caught before arthritis has developed, surgery may be performed to try to improve scaphoid healing.

A 27-year-old previously healthy man visiting the United States from Guatemala presents to the ED with acute dyspnea. He reports feeling well until about one week ago. Vital signs are BP 110/70 mm Hg, HR 120, RR 26, T 38.3°C, and pulse oximetry of 93% on room air. On exam, you note facial and lower extremity edema, hepatosplenomegaly, and lymphadenopathy. What is the most likely diagnosis? Chagas disease Dressler's syndrome Kawasaki's disease Takotsubo cardiomypathy

Correct Answer ( A ) Explanation: This patient has Chagas disease, which is caused by the parasite Trypanosoma cruzi. The parasite is transmitted through the bite of the reduviid (kissing bug) and is endemic in South and Central America. Acute myocarditis is a common complication of Chagas disease and may manifest with acute heart failure (as in this example) or arrhythmia (often refractory to rate control). Acute infection is associated with fever, edema, hepatosplenomegaly, lymphadenopathy, malaise, lymphocytosis, and elevated liver transaminases. In 25% of cases, acute infection progresses to chronic disease, typically with cardiac (dilated cardiomyopathy) or gastrointestinal (megaesophagus and megacolon) involvement. Dressler's syndrome (B) is an uncommon delayed complication occurring one week to several months after a myocardial infarction. It includes fever, malaise, pleuropericardial pain, and pleural or pericardial effusions. Kawasaki's disease (C) is seen in children younger than five years of age and results in inflammation of medium-sized vessels. It is not uncommon for the coronary arteries to be involved, increasing the risk for acute myocardial infarction. Takotsubo cardiomyopathy (D)—also known as transient apical ballooning syndrome, stress-induced cardiomyopathy, and broken-heart syndrome—is a type of nonischemic cardiomyopathy in which there is a sudden temporary weakening of the myocardium thought to be triggered by stress surge of catecholamines. Although usually benign, it can lead to acute heart failure, lethal ventricular arrhythmias, and ventricular rupture.

A new thyroid nodule is found in an 82-year-old man. He questions the risk of this nodule being malignant. You perform a detailed history and discover that his maternal uncle worked as a nuclear energy technologist and had "some kind" of head and neck cancer. As a child, the patient received radiation treatments for severe acne. He has had 6 CT scans throughout his life. He is retired, but worked as a plumber in a hydroelectric plant for 35 years. Which of the following risk factors places him at highest risk for a malignant thyroid nodule? Childhood acne treatment Exposure to CT scans Family history Occupational exposure

Correct Answer ( A ) Explanation: Thyroid cancers arise from the two distinct cell types found in the gland. The follicular cells can transform into papillary and follicular carcinomas, while the neuroendocrine calcitonin-secreting cells can transform into medullary carcinomas. Risk of malignant transformation correlates strongly with accidental radiation exposure, as in those exposed to atomic bombings (Japanese youth in Hiroshima and Nagasaki), those exposed to nuclear accidents (Chernobyl residents) and those treated with radiation therapy for the treatment of acne, ringworm or tonsillar hypertrophy (pre-1960s). Radiation exposure from routine imaging studies (B) has not been shown to increase the risk of thyroid cancer. Medullary thyroid carcinomas account for only 5-10% of all thyroid cancers. Although up to one-third of all medullary cancers are associated with a familial genetic abnormality (C), most people who develop thyroid cancer do not have an inherited condition or a family history of the disease. Working as a plumber, or working in a hydroelectric plant (D), does not constitute exposure to radiation.

Viral croup or laryngotracheitis will most commonly present with which of the following symptoms? Barking cough Drooling Dry cough of a two weeks' duration Muffled hot potato voice

Correct Answer ( A ) Explanation: Viral croup typically begins as an upper respiratory infection with coryza and a low grade fever for 12 to 72 hours. It is characterized by a barking (seal-like) cough. Other symptoms include stridor, retractions, and hoarseness.These symptoms are worse at night, during the first 24 to 48 hours and usually resolve over the course of one week. Treatment of mild croup is 0.15 mg/kg to 0.6 mg/kg oral dexamethasone as a one-time dose. If stridor is present at rest, nebulized epinephrine is an accepted treatment along with the dexamethasone. The most common viruses causing croup are parainfluenza types 1 to 3. Most episodes of croup are mild. Approximately 1 percent to 8 percent require hospitalization. Drooling (B) and a muffled hot potato voice (D) are symptoms seen more commonly in epiglottitis, a rapidly progressive, potentially life-threatening bacterial infection of the epiglottis and the surrounding structures. A dry cough of two weeks' duration (C) may be a symptom of a post-nasal drip commonly associated with seasonal allergies and allergic rhinitis.

A 15-year-old girl presents to the emergency department complaining of headaches. The patient denies fever, sore throat, vomiting, and head trauma. Physical exam reveals facial acne, papilledema, and splenomegaly. Excess of which of the following vitamins is most likely causing her symptoms? Vitamin A Vitamin B3 Vitamin B6 Vitamin C Vitamin E

Correct Answer ( A ) Explanation: Vitamin A excess can cause acute papilledema, characterized by blurred disc edges, an absent physiologic cup, and intraretinal exudates. It can also cause anorexia, slow growth, drying and cracking skin, enlargement of liver and spleen, bone fragility, swelling and pain of long bones, increased intracranial pressure, alopecia, and carotenemia. Vitamin A (retinol) is a fat-soluble vitamin and component of retinal pigments for vision in dim light. It is also involved in bone and tooth development and formation and maturation of epithelia. Vitamin A can be found in liver, fish, liver oils, whole milk, carotenoids, green vegetables, yellow fruits and vegetables. Vitamin A can be converted to retinoic acid (tretinoin) and isotretinoin, which are used in the treatment of acne and keratosis pilaris. Vitamin B3 (niacin) (B) is a constituent of coenzymes I and II; NAD, NADP cofactors in a number of dehydrogenase systems. It can be found in meat, fish, liver, grains, peanuts, cereals, and green vegetables. Niacin deficiency can cause pellagra (diarrhea, dementia, and dermatitis). Vitamin B3 excess can cause skin flushing, and itchiness. Vitamin B6 (C), also known as pyridoxine, is a constituent of coenzymes for decarboxylation, transamination, and fatty acid metabolism. Pyridoxine deficiency can cause glossitis, cheilosis, irritability, convulsions, hypochromic anemia, peripheral neuritis, and oxaluria. Vitamin B6 excess can cause sensory neuropathy. Vitamin C (D), ascorbic acid, is necessary for collagen cross-linking and can be found in citrus fruits, tomatoes, berries, cabbage, and green vegetables. Vitamin C deficiency can cause scurvy (bleeding, bruising, and poor wound healing). Vitamin C excess can cause oxaluria. Vitamin E (E) (tocopherol) is a fat-soluble vitamin that minimizes the oxidation of carotenes, vitamin A and linoleic acid, and also stabilizes membranes. Deficiency can present as reduced RBC hemolysis in premature infants and loss of neural integrity. Vitamin E excess effects are unknown.

A 12-year-old boy with Wolff-Parkinson-White syndrome presents with palpitations for the past hour. His blood pressure is 110/62 mm Hg and pulse is 166 bpm. The ECG reveals a narrow-complex tachycardia. Vagal maneuvers are ineffective. Which of the following is the next most appropriate management? Adenosine Cardioversion Procainamide Radiofrequency ablation

Correct Answer ( A ) Explanation: Wolff-Parkinson-White syndrome is a congenital cardiac defect in which an abnormal conduction pathway exists between the atria and the ventricles. It is commonly accompanied by congenital cardiomyopathy. Infantile or childhood onset of paroxysmal tachycardia is the typical manifestation. These appear as a narrow complex tachycardia and referred to as orthodromic. In orthodromic tachycardia, the normal pathway is used for ventricular depolarization, and the accessory pathway (bundle of Kent) is used for the retrograde conduction essential for reentry. Management begins with vagal maneuvers. If unsuccessful and the patient is hemodynamically stable, AV nodal blocking agents, such as adenosine or calcium channel blockers, can be administered. Any hemodynamically unstable tachycardia is treated with synchronized cardioversion (B). This patient is conversing and alert, and also has a stable blood pressure. As such, cardioversion should be delayed until the patient fails medication therapy. Antidromic (widecomplex) tachycardias associated with WPW should be treated with procainamide (C). It is important not to administer these patients AV nodal blocking agents such as calcium channel blockers since this can lead to cardiovascular collapse because all of the electrical impulses travel down the accessory pathway which does not have the same ability to delay propagation of the impulse like the AV node. Radiofrequency catheter ablation (D) is the definitive treatment of WPW syndrome, but is not recommended during the initial stabilization of an acute presentation. In those in which RF ablation is contraindicated, one may consider long-term pharmacologic treatment of WPW with verapamil, procainamide, amiodarone or sotalol.

A 65-year-old woman complains of acute pain in the left eye and vomiting after walking into a movie theater. On examination, her cornea is cloudy. She has photophobia and a fixed pupil. She reports an allergy to sulfa. What is the most appropriate action? Administration of acetazolamide Measurement of intraocular pressure Topical mydriatic agent Ultrasound of the eye

Correct Answer ( B ) Explanation: Acute angle-closure glaucoma occurs classically when patients move into a situation with lower light requiring dilation of the pupil and then obstruction of aqueous humor outflow track. Patients experience the abrupt onset of symptoms including eye pain, headache, vomiting, and blurred vision. Patients may also describe seeing a halo around lights. On examination, the conjunctiva is injected with a cloudy appearance to the cornea. The pupil is typically mid-sized or dilated and either fixed or sluggishly reactive. One must suspect the diagnosis clinically and then measure and document the intraocular pressure. Goals of treatment include rapid reduction of intraocular pressure through both topical and intravenous agents. Topical agents include beta-blockers (e.g. timolol), miotic agents (pilocarpine), and steroids. Intravenous agents for severe cases include mannitol as an osmotic diuretic and acetazolamide as a carbonic anhydrase inhibitor to decrease aqueous humor production. Definitive correction requires surgery. Administration of acetazolamide (A) is part of the standard treatment for acute angle-closure glaucoma, however, this patient reports a sulfa allergy and therefore is contraindicated. A topical mydriatic agent (C) would further dilate the pupil and worsen the symptoms. Pilocarpine is administered as a miotic agent to constrict the pupil and hopefully allow flow of the aqueous humor. An ultrasound of the eye (D) will not be helpful in the diagnosis of this condition. Ultrasound is routine in the evaluation of patients for retinal or vitreous detachment and also as a surrogate for intracranial pressure through measurement of the optic nerve diameter.

Analysis of a peripheral blood smear reveals a normocytic, normochromic anemia with blast cells and Auer rods. This is most consistent with what diagnosis? Acute Lymphoblastic Leukemia Acute Myelogenous Leukemia Chronic Lymphocytic Leukemia Chronic Myelogenous Leukemia

Correct Answer ( B ) Explanation: Acute myeloid leukemia (AML) is the most common acute leukemia in adults and comes from an excessive proliferation myeloid precursors cells. Auer rods are pathognomonic for AML. Auer rods are only found in myeloblasts, and appear as pinkish-red rod-like structures in the cytoplasm. Myeloblasts cells are immature hematopoietic cells with large nuclei and prominent nucleoli that can develop into basophils, eosinophils and neutrophils. Acute Lymphoblastic Leukemia (A) presents with normocytic, normochromic anemia and blast cells but the blast cells are lymphoid rather than myeloid and therefore do not contain auer rods. Chronic Lymphocytic Leukemia (C) may show normocytic, normochromic anemia but the smear will be dominated by smudge cells (fragile lymphocytes that smudge on slide prep). Chronic Myelogenous Leukemia (D) may show normocytic, normochromic anemia but the smear will contain the more mature myelocytes and metamyelocytes rather than myeloblasts.

A 65-year-old man presents to your office with sudden severe chest pain, asymmetric blood pressures in his arms with a new evolving aortic regurgitation murmur. The patient describes the chest pain as 10 out of 10 in severity. You decide to call 911 as you suspect what acute emergency? Acute pericarditis Aortic dissection Myocardial infarction Pulmonary embolism

Correct Answer ( B ) Explanation: Aortic dissection has a typical clinical presentation of sudden onset of severe chest pain often described as sharp, tearing, or ripping. Pain can be anterior with ascending aortic dissection or back pain with descending aortic dissection. Pulse and blood pressure differentials commonly can be caused by partial compression of subclavian arteries. Most patients present with markedly elevated blood pressure. Hypotension can indicate hemorrhage, cardiac tamponade, or severe aortic regurgitation. Risk factors include chronic hypertension, atherosclerosis, family history of aortic aneurysms, trauma, collagen disorders, bicuspid aortic valve, aortic coarctation, vasculitis, Turner's syndrome and cocaine abuse. Acute pericarditis (A) usually has severe constant pain that localizes over the anterior chest and may radiate to arms and back; it can be differentiated from myocardial ischemia because the pain intensifies with inspiration and is relieved by sitting up and leaning forward (the pain of myocardial ischemia is typically not pleuritic). On exam, there may be a pericardial friction rub that is best heard with the patient upright and leaning forward and by pressing the stethoscope firmly against the chest. The pain of myocardial infarction (C) usually has a crescendo-like onset and is not as intense and does not have pulse differentials. In pulmonary embolism (D) the most common symptom is dyspnea with or without pleuritic chest pain and can be accompanied with fever and a cough with a cardiac exam that includes tachycardia, increased pulmonic component of S2, murmur of tricuspid insufficiency, RV heave and right-sided S3.

A patient with chronic spinal pain begins to obtain off-the-street opioids in an attempt to enhance his prescribed opioid analgesics. In addition to psychotherapy for substance dependence, you prescribe which of the following substitution adjuvant medications? Bromocriptine Buprenorphine and naloxone Disulfiram Modafinil

Correct Answer ( B ) Explanation: Dependence can be defined as the state at which a person who uses substances develops tolerance or withdrawal symptoms (physical or psychological or both). Physical dependence can occur from recreational, illicit, or prescription drug. Once a patient develops withdrawal symptoms after stopping use of an illegal or prescribed drug, substance dependence can be diagnosed. Some consider drug addiction to be dependence in the setting of significant impairment of a person's social, vocational and avocational responsibilities or in the setting of continued physical harm. Several symptoms may arise, some of the more common ones are diaphoresis, hypertension, tachycardia, tremors, confusion, hallucinations and seizures. Treatment of dependence is usually accomplished by a slow taper over weeks to months.. The use of substitute drugs that are partial agonists or less harmful are frequently used to assist dependence management. Buprenorphine/naloxone combination is used in the treatment of opioid dependence. The goal of this is to provide a clinically supervised, stable dose of a partial opioid agonist (buprenorphine) and opioid-antagonist (naloxone) to provide pain and craving control, which allows the addict to function while reducing the negative aspects of addiction. Bromocriptine (A) is used for cocaine dependence. Disulfiram (C) is used in alcohol, not opioid, dependence. Modafinil (D) shows some promise in the treatment of stimulant, not opioid, dependence.

A 26-year-old woman presents with the chief complaint of intensely itchy hands and fingers. Your exam reveals multiple clear vesicle at the palms and sides of her fingers with a tapioca-grain appearance. Unroofing the vesicles and performing a KOH preparation does not reveal hyphae. In addition to a topical corticosteroid, which of the following pieces of patient education should be given to this patient? Adhere to strict avoidance of gluten Avoid prolonged exposure of the hands to water Limit use of moisturizers as much as tolerated Use prescription-strength high-SPF lotion with UVA/UVB coverage on the hands daily

Correct Answer ( B ) Explanation: Vesiculobullous hand eczema, or dyshidrosis, should be managed with topical corticosteroids for acute flares and by avoidance of long exposure of the hands to water. Dyshidrosis is a very common form of hand dermatitis that usually presents in adults with a history of atopy and persists for life. On exam, patients will have vesicles of 1-2 mm on their soles, palms and sides of the fingers that appear to contain "grains of tapioca." Intense pruritus is common. As flares subside, patients will be left with scaling and fissuring over the affected area. A diagnosis of dyshidrosis requires ruling out other causes of hand dermatitis, and should include a KOH preparation of unroofed vesicles to rule out bullous tinea or inflammatory tinea pedis when the feet are affected. Topical corticosteroids often bring dramatic improvement in dyshidrosis during acute vesicle eruptions as well as for the fissuring that usually follows resolution of the vesicles. Patient education to wear cotton-lined vinyl gloves when working in water and to consistently apply moisturizers after washing hands is imperative in the management of this disease. Adhering to strict avoidance of gluten (A) is not useful for patients with dyshidrosis. However, this often brings symptom resolution for patients with dermatitis herpetiformis secondary to Celiac disease. Limiting use of moisturizers as much as tolerated (C) would not be beneficial to patients with dyshidrosis, and may even be harmful in preventing future flares. Use of a high-SPF with UVA/UVB coverage on the hands daily (D) would not be harmful to the patient, but is not necessary to heal acute flares or prevent reoccurrences. Most patients would benefit adequately from a sunscreen with SPF 15-40 daily. However, higher protection is useful in certain photosensitive condition, including discoid and subcutaneous lupus erythematous.

A 28-year-old previously healthy woman presents to your office with a complaint of feeling depressed. In the past 6 months she feels very tired despite sleeping 10-12 hours per night, has no desire to exercise like she did previously, has lost 10 pounds and complains of not having an appetite. She admits to having thoughts of wishing she weren't alive anymore, although denies being suicidal. Which of the following is considered first-line therapy for this patient? Amitriptyline Citalopram Olanzapine Phenelzine

Correct Answer ( B ) Explanation: Depression is a very common mental health complaint seen in primary care. Diagnosis of depression is made when patients have at least one episode of major depression and no history of mania or hypomania. An episode of major depression lasts at least two weeks and includes having five or more symptoms of depression including anhedonia, depressed mood, change in appetite or weight, insomnia or hypersomnia, low energy, inability to concentrate, feeling guilty or worthless, and thoughts of death or suicide. Selective serotonin reuptake inhibitors (SSRIs), such as citalopram, are considered first-line pharmacologic treatment for adults with depression. Initial treatment for patients with depression should include both psychotherapy and pharmacotherapy with SSRIs. Amitriptyline (A) is a tricyclic antidepressant. Tricyclic antidepressants were considered first-line before SSRIs were developed and are still used to treat depression as well as other mental health disorders and chronic pain. Olanzapine (C) is an atypical antipsychotic and is not used as first-line pharmacotherapy in the treatment of depression. Phenelzine (D) is a monoamine oxidase inhibitor and is not used as first-line treatment for depression due to having an extensive side effect profile, dietary restrictions and numerous drug-drug interactions. Monoamine oxidase inhibitors do play a role in treating atypical depression or depression that is resistant to treatment.

Which of the following medications most commonly leads to recurrent episodes of hypoglycemia? Glargine insulin Glimepiride Metformin Pioglitazone

Correct Answer ( B ) Explanation: Glimepiride is an oral hypoglycemic agent in the sulfonylurea class of drugs and has a prolonged half-life placing patients at risk of recurrent hypoglycemic episodes. Sulfonylureas act by stimulating pancreatic islet beta cells to secrete more insulin. Additionally, they increase the sensitivity of peripheral tissues to insulin. Because of their long duration of action, these drugs have been associated with recurrent and delayed hypoglycemia up to 24 hours post ingestion. Hypoglycemia is common in diabetics but is associated with significant morbidity and mortality. Recurrent hypoglycemic episodes in patients on sulfonylureas should be treated with octreotide, a somatostatin analog. Octreotide inhibits glucose stimulated insulin release thus preventing recurrent hypoglycemia. Glargine insulin (A) is a long acting insulin that provides basal insulin levels in diabetic patients. It does not have a peak activity and thus, is rarely associated with recurrent hypoglycemic episodes. Metformin (C) is in the biguanide class of oral diabetic medications. It acts by increasing peripheral sensitivity to insulin and suppresses gluconeogenesis. Metformin rarely causes hypoglycemia. Pioglitazone (D) is in the thiazolidinedione class of drugs, which act by decreasing insulin resistance. It does not cause hypoglycemic episodes.

A 67-year-old woman with New York Heart Association class III congestive heart failure presents to clinic for a routine examination. Her chief complaint is worsening peripheral edema. Which of the following abnormalities would you most expect to find during a physical examination? Finger-nail clubbing Jugular venous distension Pancreatic duct congestion Pulsus paradoxus

Correct Answer ( B ) Explanation: Heart failure takes on many forms, however, the overall pathology is a failure to pump blood forward at a sufficient rate. Etiologies include ischemic heart disease (coronary atherosclerotic disease) and cardiomyopathy. Cardiac failure can be backward, or "congestive", versus forward, or "impaired perfusion". Impaired perfusion symptoms include fatigue, weakness, poor appetite, mental status changes and exercise intolerance. Left-sided congestive failure results in dyspnea, orthopnea, paroxysmal nocturnal dyspnea, pulmonary edema, crackles and pulmonary basilar dullness to percussion. Right-sided failure causes peripheral edema, RUQ discomfort, bloating, ascites, hepatosplenomegaly, hepatojugular reflux, jugular venous distension and increased jugular venous pulsation. Congestive failure is also associated with an S3 heart sound. Finger-nail clubbing (A) is more indicative of pulmonary, instead of cardiac, pathology. Pancreatic duct congestion (C) is not associated with heart failure. Backup of the hepatic vasculature is however. Pulsus paradoxus (D) is a systolic blood pressure drop of ≥ 10 mm Hg during inspiration. It is commonly seen in pericardial tamponade, but not congestive heart failure.

Which of the following clinical scenarios can be defined as a hypertensive emergency? A 25-year-old pregnant woman in her second trimester with a blood pressure of 155/100 mm Hg with a normal urinalysis A 55-year-old man with a blood pressure of 185/90 mm Hg whose creatinine has increased from 1.0 to 2.5 mg/dL within 36 hours A 59-year-old asymptomatic man requesting a medication refill and is found to have a blood pressure of 210/110 mm Hg and an ECG consistent with left ventricular hypertrophy (LVH) A 63-year-old woman with a history of poorly controlled hypertension who presents with a finger laceration and is noted to have a blood pressure of 200/105 mm Hg

Correct Answer ( B ) Explanation: Hypertensive emergency is generally defined as a markedly elevated blood pressure in the setting of acute end-organ damage of the cardiovascular, neurologic, or renal organ system. This condition is a true medical emergency and warrants early reduction of blood pressure (preferably within one hour of identification of the condition) with titratable intravenous medications. It is important to understand, however, that an elevated blood pressure in response to an acute condition is often physiologic; aggressive lowering of the pressure in these conditions (e.g., ischemic stroke) may actually increase morbidity and mortality. Renal failure can be seen as both a consequence and cause of hypertension. Uncontrolled hypertension may cause acute kidney injury and can accelerate the progression of injury in patients with chronic renal failure. Acute worsening of kidney function as seen in this patient whose creatinine increased acutely from 1 to 2.5 mg/dL—in the setting of elevated blood pressure—should be considered a hypertensive emergency and warrants immediate treatment. Hypertension is one of the most common complications in pregnancy. Hypertension in pregnancy is defined as a systolic pressure > 140 mm Hg or a diastolic pressure > 90 mm Hg. Severe hypertension is classified as a systolic pressure > 160 mm Hg or diastolic > 105 mm Hg. Pre-eclampsia is defined as hypertension occurring after 20 weeks gestation with proteinuria or any signs or symptoms of end-organ damage (e.g., elevated LFTs) and should be considered a hypertensive emergency. Hypertension occurring after 20 weeks gestation without these signs or symptoms (A) is termed gestational hypertension. Patients who are hypertensive, but asymptomatic, and show no evidence of acute end-organ damage (C and D) in most cases, do not need acute lowering of their blood pressure. Patients with chronic hypertension may have an altered autoregulatory range; rapid normalization of their elevated blood pressure may in fact lead to hypoperfusion and ischemia. In response to chronic hypertension, the heart is remodeled in a cycle starting with increased wall stress that leads to hypertrophy and impaired diastolic function. When LVH is present in the setting of hypertension, early follow-up should be arranged because such patients are at increased risk for MI, heart failure, stroke, and sudden death.

A 2-week-old baby boy is brought to clinic for routine care by his mother who is concerned about sudden infant death syndrome. Which of the following is a level A recommendation from the American Academy of Pediatrics to help reduce the risk of sudden infant death syndrome? Cardiorespiratory monitoring when the baby is sleeping Offering a pacifier at nap time and bedtime Padded crib bumpers Side sleep position

Correct Answer ( B ) Explanation: Offering a pacifier at naptime and bedtime is a level A recommendations by the American Academy of Pediatrics (AAP) that helps to reduce the risk for sudden infant death syndrome. Other recommendations include supine sleep position, use of a firm sleep surface, room-sharing without bed-sharing, keeping soft objects out of the crib, avoiding smoke exposure, breastfeeding, avoiding overheating. Sudden unexpected death of infancy (SUDI) describes infant deaths that cannot otherwise be explained. SIDS peaks between 1 and 4 months with 90% of cases occurring prior to 6 months. Risk factors for SUDI include infant co-sleeping, prone and side sleep positions, parental obesity, sleeping on a soft surface, overheating, and second-hand smoke exposure. Cardiorespiratory monitoring (A) is not recommended by the AAP for reducing the risk of SIDS. Padded crib bumpers (C) should be avoided as well as loose bedding and soft objects such as stuffed animals. Side sleep position (D) has an odds ratio of 2.0-8.7 for SIDS as compared with supine sleeping.

A 24-year-old woman presents to the clinic with mild otalgia and a chronic feeling that her right ear is "blocked." An otoscopic examination reveals a marginal tympanic membrane perforation. Keratin debris is prominent in this area. Which of the following risk factors likely contributed to the development of this patient's condition? Diabetes mellitus Prolonged eustachian tube dysfunction Recent upper respiratory infection Recent water exposure

Correct Answer ( B ) Explanation: Prolonged eustachian tube dysfunction is the most common cause of cholesteatoma formation, as the negative middle ear pressure draws the upper, flaccid portion of the tympanic membrane inward. The result is the formation of a squamous epithelium-filled sac which can fill with desquamated keratin and cause chronic infection. Cholesteatomas pose a significant danger to patients as they can eventually erode through the mastoid bone and destroy the ossicular chain. The inner ear and facial nerve may also become eroded, and intracranial involvement occasionally occurs. Because of the risk of devastating erosion, all patients with cholesteatomas should be referred to ENT for surgical marsupialization of the sac or its complete removal. Removal of the cholesteatoma may take precedence over preserving auditory function. Cholesteatomas should be suspected in patients with chronic mild otalgia, hearing loss, or chronic ear fullness, particularly with a long history of eustachian tube dysfunction. Diabetes mellitus (A) is not a risk factor for cholesteatoma formation. It is a risk factor for malignant otitis externa, which may cause infectious osteomyelitis to spread from the floor of the ear canal to other regions of the ear and skull. Recent upper respiratory infections (C) are unlikely to cause a cholesteatoma. They may, however, contribute to development of acute otitis media. Recent water exposure (D) does not contribute to cholesteatoma formation. It is more likely to cause acute otitis externa.

Which of the following historical or physical exam findings is most consistent with a diagnosis of hypomagnesemia? Bradycardia Hyperreflexia Hypotonia Perioral paresthesias

Correct Answer ( B ) Explanation: Symptoms related to hypomagnesemia typically involve the neuromuscular and cardiovascular systems. It is also important to note that magnesium depletion is often seen in conjunction with other electrolyte abnormalities, most notably hypokalemia, which can contribute to the patient's presentation. Common manifestations of hypomagnesemia include muscle cramping, weakness, hyperreflexia, ataxia, and seizures. Electrocardiogram findings include QT prolongation, premature ventricular contractions, and tachydysrhythmias such as atrial fibrillation, multifocal atrial tachycardia, ventricular tachycardia or fibrillation, and torsades de pointes. Although symptoms are usually apparent when serum levels are < 1.2 mEq/L, the serum level alone may not accurately reflect the total body magnesium depletion as the majority of magnesium is intracellular. Mild symptoms can be treated with oral supplementation, whereas rapid parenteral replacement is indicated in cases of cardiac instability or severe symptoms. Bradycardia (A) is not typically seen in hypomagnesemia. Patients often have muscle cramping and tetany, not hypotonia (C), which can be seen with hypercalcemia. Perioral paresthesias (D) can be a manifestation of hypocalcemia.

An 84-year-old woman with new onset altered mental status is sent to the ED from her assisted living facility. According to the facility staff, she has had a productive cough for two days. Her vital signs are blood pressure 90/60 mm Hg, heart rate 92 beats per minute, respiratory rate 25 breaths per minute, temperature 38.3°C, and oxygen saturation 95% on room air. Laboratory results reveal a WBC of 11,000. A chest radiograph shows a right lower lobe infiltrate. Which aspect of this patient's presentation is consistent with systemic inflammatory response syndrome? Blood pressure Heart rate Infiltrate on chest radiograph White blood cell count

Correct Answer ( B ) Explanation: Systemic inflammatory response syndrome (SIRS) is a state of systemic inflammation related to cytokine storm. Sepsis, pancreatitis, burns, trauma, and a number of other conditions can cause it. SIRS is defined by meeting two or more of the following criteria: The patient's heart rate is 92 beats per minute and therefore meets SIRS criteria (> 90). In addition, this patient's respiratory rate is 25 breaths per minute and temperature is 38.3°C, which also meets SIRS criteria. Given that she meets two criteria, she can be classified as having at least SIRS. In addition, her chest radiograph and clinical presentation are consistent with pneumonia. Therefore, in light of meeting SIRS criteria and having a suspected infection, this patient can be classified as having sepsis. Blood pressure (A) and chest radiograph findings (C) are not part of the SIRS criteria. The patient's WBC (D) does not meet the WBC criteria for SIRS (< 4000 or > 12,000 or > 10% bands on the cell differential).

Tonya is a 35-year-old nurse who presented to the emergency department last week for a neurologic event. She had fatigue, generalized dull aching, decreased color vision in her left eye, and right arm and leg weakness that lasted for 48 hours. A magnetic resonance imaging study of the brain showed no evidence of a stroke but revealed scattered T2 lesions. She has never had these symptoms before. What is the treatment for this acute episode? Glatiramer High-dose corticosteroids Interferon beta Low-dose corticosteroids

Correct Answer ( B ) Explanation: This woman presents with the classic signs and symptoms of multiple sclerosis. In most cases, the disease follows a relapsing-remitting pattern, with short-term episodes of neurologic deficits that resolve completely or almost completely. A minority of patients experience steadily progressive neurologic deterioration. Typical symptoms include sensory disturbances, motor weakness, optic neuritis, impaired coordination, and fatigue. Multiple sclerosis is a clinical diagnosis. Two neurologic deficits lasting for at least 24 hours, separated in space and time are considered diagnostic. Magnetic resonance imaging is highly sensitive for revealing sharply delineated regions of demyelination throughout the central nervous system white matter, especially in periventricular areas, and can be used to diagnose multiple sclerosis in cases not meeting the threshold for clinical diagnosis. Cerebrospinal fluid is also obtained for diagnostic purposes. The analysis typically demonstrates oligoclonal bands and an increased immunoglobulin G concentration. Exacerbations affect 85% of patients with multiple sclerosis; infections and stress may play a role. For those with significant, acute symptoms, corticosteroids are the treatment of choice. The initial drug therapy for acute episodes, whether CIS or a subsequent acute episode, is intravenous corticosteroids. Corticosteroids decrease T-cell response to antigens, production of cytokines, secretion of immunoglobulins, and levels of myelin basic protein in cerebrospinal fluid. Treatment with high-dose intravenous methylprednisolone (more than 500 mg/day for at least 3 days) is a typical recommended treatment. Disease-modifying agents should be initiated early in the treatment of multiple sclerosis to forestall disease and preserve function. Symptom management constitutes a large part of care; neurogenic bladder and bowel, sexual dysfunction, pain, spasticity, and fatigue are best treated with a multidisciplinary approach to improve quality of life. Interferon beta (C) or glatiramer (A) should be used as the first disease-modifying therapy, and treatment should be initiated as soon as possible after diagnosis in patients. Patients who start on interferon beta and either cannot tolerate treatment or for whom the treatment is not effective are typically changed to glatiramer and vice versa. Low-dose corticosteroids (D) are not an indicated treatment for patients with multiple sclerosis.

A 72-year-old man presents for evaluation of palpitations. He has a regular, wide complex tachycardia at a rate of 140. Which of the following supports a diagnosis of ventricular tachycardia? Discordance of the QRS axis in the precordial leads Fusion beats Leftward axis ST elevation greater than 5 mm

Correct Answer ( B ) Explanation: Ventricular tachycardia (VT) originates from a location within the ventricle typically outside of the normal conduction system. Increased automaticity or a reentry circuit may cause it. The majority of patients with VT have underlying cardiac disease. The most common form of VT is monomorphic where the QRS complexes appear the same morphologically. Since the electrical impulse originates outside of the conduction system, the QRS complex is wide. It is sometimes challenging to differentiate between VT and other wide complex tachycardias (eg, supraventricular tachycardia with aberrancy). VT is typically regular although there may be a small amount of irregularity helping to distinguish between it and supraventricular tachycardia. Fusion beats occur when impulses from two different locations (one within the ventricle and one in a supraventricular location) activate the ventricle. The result is a QRS complex with morphology resembling a hybrid of a sinus beat and intraventricular beat. These are diagnostic of VT because they represent AV dissociation. Capture beats occur when a sinus beat is normally conducted and a single beat with the sinus QRS morphology occurs within a wide complex tachycardia. Discordance of the QRS axis in the precordial leads (A) is NOT suggestive of VT. Typically across the precordium in VT the QRS complex demonstrates concordance or its polarity. A leftward axis (C) is not suggestive of VT and a more characteristic axis deviation is right superior. ST elevation greater than 5 mm (D) is not part any diagnostic criteria for the identification of VT.

An elderly man with diabetes mellitus suffers a myocardial infarction, is admitted to the intensive care unit and subsequently stabilizes. The next day he undergoes coronary angiography with iodinated contrast. 20-hours after this test, the nursing staff calls with a concern of decreased urinary output. His serum creatinine on admission was 1.1 mg/dL, but a stat lab order reveals a current serum creatinine of 1.9 mg/dL. The risk of developing this clinical scenario could have been reduced with pre-angiography treatment consisting of intravenous hydration and which of the following medications? Deferoxamine Dimercaprol N-acetylcysteine Penicillamine

Correct Answer ( C ) Explanation: Acute kidney injury (AKI), also known as acute renal failure (ARF), is defined as an increase in creatinine >0.3 mg/dL over a 48 hour span, a >50% increase in baseline creatinine or a decreased urinary output <0.5 ml/kg/hr for more than 6 hours. There are multiple prerenal, renal and postrenal causes. One of the more common intrinsic, or renal, causes is contrast-induced acute kidney injury, occurring after intravenous delivery of iodinated contrast medium. Those with underlying chronic renal disease, congestive heart failure, and diabetes mellitus are at higher risk. This condition commonly manifests as decreased urinary output and an increase in creatinine 25% over baseline or >0.5 mg/dL within 48 hours of a procedure which used iodinated contrast. To decrease the overall risk of contrast-induced-nephropathy, the clinician should practice the following: maximize hemodynamic stability with intravenous saline prehydration and posthydration, minimize contrast volume, consider iso-osmolar contrast, consider pre and post hemofiltration and hold diuretics, NSAIDs and ACE-inhibitors. Several drugs are also added to hydration therapy, such as N-acetylcysteine, vasodilators, ascorbic acid, theophylline and aminophylline. Deferoxamine (A) is a chelating agent used in treating hemochromatosis (iron overload) and iron poisoning. Dimercaprol (B) is a chelator used in arsenic, mercury and lead poisoning. Penicillamine (D) is a copper chelating agent used in the treatment of Wilson's disease.

Which of the following is consistent with adrenal insufficiency? Hyperchloremic metabolic acidosis Hyperkalemia, hyperphosphatemia and hypocalcemia Hyponatremia, hyperkalemia Severe hypocalcemia

Correct Answer ( C ) Explanation: Adrenal insufficiency leads to decreased levels of mineralocorticoids resulting in hyponatremia and hyperkalemia. Adrenal insufficiency can result from primary adrenal failure or secondary adrenal disease from dysfunction of the hypothalamic-pituitary-adrenal (HPA) axis. Patients will present with nonspecific symptoms including fatigue, nausea, abdominal cramping, dizziness and weakness. In patients with primary adrenal insufficiency, skin hyperpigmentation from increased ACTH secretion is common. Due to the absence of mineralocorticoids, patients will have salt wasting leading to hyponatemia and hyperkalemia. Metabolic acidosis is common in presentation as well. Patients with adrenal insufficiency require daily hydrocortisone and fludrocortisone therapy. When patients present with critical illness, they should be given stress dose hydrocortisone (100 mg). Hyperchloremic metabolic acidosis (A) is associated with renal tubular acidosis (RTA), renal failure, and gastrointestinal loss of bicarbonate with diarrhea. Hyperkalemia, hyperphosphatemia and hypocalcemia (B) are seen in tumor lysis syndrome. Severe hypocalcemia (D) is seen after parathyroidectomy.

Which of the following is a known precipitant of aplastic crisis in patients with sickle cell disease? Coxsackie virus Parainfluenza virus Parvovirus B19 Respiratory syncytial virus

Correct Answer ( C ) Explanation: An aplastic crisis occurs when there is a decline in the production of red blood cells. Parvovirus B-19 has been implicated in causing a transient aplastic crisis in patients with sickle cell disease. Patients infected with parvovirus B19 can be asymptomatic, have only mild upper respiratory symptoms, or present classically with fever, headache, nausea, and an erythematous rash on the cheeks followed by a diffuse maculopapular rash characteristic of erythema infectiosum. An aplastic crisis can occur regardless of the severity of symptoms related to the parvovirus infection. Patients will present with worsening fatigue and pallor. Laboratory studies show a markedly decreased hemoglobin in conjunction with a decreased reticulocyte count, often < 0.5%. As the bone marrow recovers, usually in 1-2 weeks, the reticulocyte count will be elevated above normal. Treatment includes a red blood cell transfusion and close follow up. Coxsackie virus (A) causes hand, foot, and mouth disease. Croup is most often caused by parainfluenza virus (B). Respiratory syncytial virus (D) causes bronchiolitis in infants. None of these viruses has been known to cause aplastic crisis.

You are treating a patient with type 2 diabetes mellitus with three different classes of medications. Which of the following medications is proven to decrease mortality in this disease? Glyburide Insulin Metformin Pioglitazone

Correct Answer ( C ) Explanation: Type 2 diabetes mellitus (DM2) affects approximately 20 million Americans and is the sixth leading cause of death in the USA. It is the most common cause of renal failure and blindness. Three main goals of treatment are lifestyle modifications, management of cardiovascular risk factors, and normalization of hyperglycemia. Lifestyle modifications include weight loss, low fat reduced calorie diet and 150 minutes per week of physical activity. Aspirin, statins and ACE-inhibitors are used to modify the cardiovascular risk factors of hypertension, dyslipidemia and microalbuminuria. DM2 is characterized by insulin resistance, relative insulin deficiency (decreased secretion) and increased hepatic glucose production. As such, there are several different classes of medications used to treat DM2. However, metformin, a first-line agent, is the only oral DM2 medication proven to reduce mortality. Glyburide (A) is one of the sulfonylurea medications that stimulates the pancreas to secrete more insulin. However, it has not been shown to decrease mortality. Insulin (B) and pioglitazone (D) have also not been shown to reduce mortality.

A woman suffers from an acute attack of vertigo, nausea, and vomiting. You suspect viral labyrinthitis. Which of the following medications is the best choice in treating the vertigo? Acyclovir Phentermine Prednisolone Prochlorperazine

Correct Answer ( C ) Explanation: Commonly a complication of otitis media, upper respiratory infection or meningitis, labyrinthitis is an abnormality of the inner ear vestibular system, which consists of three semicircular canals, vestibule, utricle and sacule. When the vestibular system is damaged, injured or inflamed, balance deficits and vertigo arise. However, since an anatomic connection between the vestibular system and cochlea exist, tinnitus and hearing impairment also occur. The classic clinical picture of labyrinthitis is acute, profound, incapacitating vertigo, with nausea, vomiting and nystagmus. Tinnitus and hearing impairment usually accompany the onset of vertigo. These symptoms resemble those of Meniere's disease, however in Meniere's disease, symptoms are intermittent and possibly less severe. Furthermore, the pathology is different between the two conditions. Labyrinthitis is mainly due to inflammation, while an excess of inner ear fluid causes Meniere's disease. Treatment of suspected viral labyrinthitis includes bed rest, hydration, antiemetics and vestibular depressants. Treatment with corticosteroids during the acute period of vertigo has been shown to improve the recovery of peripheral vestibular function in patients with acute labyrinthitis. Benzodiazepines are also effective. Currently, the role of antiviral therapy, like acyclovir (A), in the treatment of labyrinthitis is not established. Studies to date do not show an improvement over placebo. Phentermine (B) is used as an appetite suppressant and plays no role in the treatment of labyrinthitis. Prochlorperazine (D), an antiemetic, is commonly used in the treatment of acute viral labyrinthitis. Its dopamine-antagonism and anticholinergic effects help relieve nausea and vomiting, but do not diminish the distressing sensation of vertigo.

Which of the following drugs is classified as a calcium channel blocker? Alendronate Calcitonin Nicardipine Olanzapine

Correct Answer ( C ) Explanation: Nicardipine is a dihydropyridine calcium-channel blocking agent used for the treatment of vascular disorders such as chronic stable angina, hypertension, and Raynaud's phenomenon. It is available in oral and intravenous formulations. Its mechanism of action and clinical effects closely resemble those of nifedipine and the other dihydropyridines (amlodipine, felodipine), except that nicardipine is more selective for cerebral and coronary blood vessels. Furthermore, nicardipine does not intrinsically decrease myocardial contractility and may be useful in the management of congestive heart failure. Nicardipine also has a longer half-life than nifedipine. Alendronate (A) is a bisphosphonate that acts as an osteoclast inhibitor to inhibit bone resorption. Calcitonin (B) works to lower serum calcium concentration by inhibiting calcium absorption by the intestines, inhibiting osteoclast activity in the bones, and inhibits renal tubular cell reabsorption of calcium allowing it to be excreted in the urine. Olanzapine (D) is an atypical antipsychotic that is used for the treatment of schizophrenia and bipolar disorder. Olanzapine is structurally similar to clozapine and quetiapine, but is classified as a thienobenzodiazepine.

You suspect postconcussive syndrome in a previously healthy young woman who was involved in a motor vehicle collision last week. Initial neuroimaging and neurologic examination were normal. Emergency department management was mainly supportive. Currently, she complains of memory and concentration difficulties. Neurologic examination is normal. Which of the following would you recommend at this time? Lumbar puncture Noncontrast MRI Referral to a cognitive therapist Repeat CT scan

Correct Answer ( C ) Explanation: Postconcussive syndrome is a constellation of symptoms which occur after head trauma. The underlying pathology is based on axonal injury of the cerebrum. Psychological abnormalities also play a role in the development of this syndrome. There are several different definitions, however, most agree on the following: minor head injury (GCS 13-15), no CT scan abnormalities, and at least 3 of the following symptoms which begin within several weeks post injury: headache, dizziness, irritability, fatigue, insomnia, memory deficits, concentration deficits, photophobia or phonophobia. Other symptoms include attention deficits, delayed reaction time, and decreased appetite and libido and personality changes. Neurological exam is normal. No treatment has been proven beneficial, but analgesics, antiemetics, psychotherapy, cognitive therapy, vocation counseling and stress management are frequently recommended. Some of the best evidence points toward patient education, reassurance and regular follow-up as being effective. Although MRI (B) is more sensitive than CT in finding abnormalities in patients with postconcussive syndrome, the abnormalities found pose little clinical significance and rarely change treatment or outcome. Lumbar puncture (A) is utilized when there is a suspicion of meningitis or subarachnoid hemorrhage. It has no role in postconcussive syndrome. Repeat CT scanning (D) is largely noncontributory, unless a patient presents with focal neurological signs or is at risk of delayed-hemorrhage, as in elderly anticoagulated patients.

Concerning the hearing conduction pathway, which of the following is the correct passage of sound waves as they travel from the external auditory canal to the pons? Ossicles -> tympanic membrane -> oval window -> vestibulocochlear nerve -> cochlea Ossicles -> tympanic membrane -> round window -> cochlea -> vestibulocochlear nerve Tympanic membrane -> ossicles -> oval window -> cochlea -> vestibulocochlear nerve Tympanic membrane -> ossicles -> round window -> vestibulocochlear nerve -> cochlea

Correct Answer ( C ) Explanation: Proper hearing relies on conduction of air waves thru the external auditory canal, tympanic membrane, middle ear ossicles (malleus, incus, stapes) and oval window. From there, waves enter the inner ear cochlea, and pass thru the "outer tubes", the perilymph-containing scala vestibuli (and its vestibular membrane) and scala tympani (and its basilar membrane). Waves then pass into the cochlear "inner tube", the endolymph-containing scala media (or cochlear duct). This causes vibrations of the tectoral membrane and organ of Corti, thus transforming mechanical wave energy into electrical nerve signals, which are transported out of the inner ear via the cochlear division of cranial nerve 8, the vestibulocochlear nerve. From the vestibule, the vestibular division of CN8 joins the cochlear division to form the vestibulocochlear nerve proper, which travels through the internal acoustic meatus of the petrous part of the skull's temporal bone to end at the pons. After the external auditory canal, sound waves hit the tympanic membrane (eardrum) before, not after, the ossicles (A and B). After the ossicles, sound waves are transmitted into the cochlea thru the oval, and not round (D), window.

A young woman with dyspnea on exertion presents for evaluation. She is subsequently diagnosed with sarcoidosis. Which of the following would you expect to appear in her lymph node biopsy report? Acid-fast organisms Congo red stain birefringence Noncaseating granulomas Reed-Sternberg cells

Correct Answer ( C ) Explanation: Sarcoidosis is an idiopathic widespread inflammatory condition. It is distinguished by accumulation of T cells and mononuclear phagocytes in affected organs, along with noncaseating granulomas and disturbance in normal tissue architecture. The lung is the most commonly involved organ, with skin, eye, liver, and lymph node involvement also common. The disorder can be acute, subacute and self-limiting, or a chronic waxing and waning disease occurring over many years. In the U.S.A., African-Americans are affected 10x more than Caucasians. Patients may be asymptomatic or complain of fatigue, malaise, weight loss, and fever. Commonly, patients present with symptoms based on the involved organ. For the lungs, this may include exertional dyspnea; dry, nonproductive cough; and vague chest pain. Exam findings are usually normal but may sometimes include crackles. Lymphadenopathy may be present. Chest X-ray is classic for bilateral hilar lymphadenopathy. Diagnosis is made by fiberoptic bronchoscopy with transbronchial biopsy showing a sarcoid granuloma, which is a well-formed noncaseating epithelioid cell granuloma surrounded by a rim of fibroblasts and lymphocytes. Management includes systemic corticosteroids as first-line therapy with methotrexate as an alternative. Acid-fast organisms (A), such as Mycobacterium species, are present in pulmonary tissues of patients with tuberculosis, not pulmonary sarcoidosis. In the presence of amyloid fibrils, congo red stain will appear as green birefringence (B) under polarized light. This is a diagnostic test for amyloidosis, not sarcoidosis. Reed-Sternberg cells (D) are found in Hodgkin lymphoma, not sarcoidosis.

A 28-year-old man presents with scrotal swelling and pain. Examination reveals a left, nontender, swollen spermatic cord that feels like a "bag of worms." The swelling reduces when the patient moves to a supine position. There are no skin lesions, hernia or testicular abnormalities. In addition to scrotal ultrasonography, you may consider ordering further imaging with attention to which of the following organs? Ascending colon Descending colon Kidney Urinary bladder

Correct Answer ( C ) Explanation: Scrotal mass and pain are common complaints in the primary care office. Etiologies range from incidental benign conditions to serious cancers to surgical emergencies. Any patient reporting scrotal swelling needs to be evaluated immediately. A careful history and physical examination, including transillumination, is necessary. The evaluation begins with determining which structure is swollen: skin, epididymis, testicle or spermatic cord. Dilation of the pampiniform venous plexus along the spermatic cord is referred to as varicocele. This condition is suspected in any swelling of the spermatic cord, especially when palpation reveals a nontender "bag or worms" consistency. There are primary and secondary types of varicocele. Primary varicocele is usually idiopathic in nature. Secondary varicocele can occur due to abdominal mass compression of the renal veins (more common with right sided varicocele), or superior mesenteric artery compression of the left renal vein (aka "Nutcracker Syndrome"; most commonly due to renal cell carcinoma and retroperitoneal fibrosis/adhesions). Patients with varicocele usually complain of scrotal heaviness and a "dragging" aching pain. It mostly occurs in tall, thin men between the ages of 15 and 30 years, and affects the left side more than the right. The "wormy" mass or swelling typically reduces when the patient is supine. Varicoceles, although not life threatening may cause decreased testicular artery flow and infertility, and therefore should be evaluated further with ultrasonography and possibly abdominal imaging. Colonic (A and B) and bladder (D) masses are unlikely to cause venous compression and subsequent varicocele.

A 23-year-old woman presents with acute onset dizziness and palpitations. Her ECG reveals evidence of AV nodal reentrant tachycardia with a rate of 170. Her blood pressure is 140/70 mm Hg. Which of the following is the most appropriate initial treatment? Atropine Cardioversion Valsalva maneuver Verapamil

Correct Answer ( C ) Explanation: Supraventricular tachycardias (SVT) include paroxysmal, reentry or preexcitation tachycardias. Reentry SVTs include AV nodal reentry (AVNRT), atrioventricular reentry, or atrial reentry. Reentry circuits require the presence of at least two different conduction pathways with differential refractory times. It is characterized by an abrupt onset and termination of tachycardia, that distinguishes it from sinus tachycardia, which has gradual changes in rate. It is precipitated by a premature atrial or ventricular contraction or hyperadrenergic state. The ECG shows a regular, fast rhythm with absent P waves and narrow QRS complex. Unstable patients require immediate synchronized cardioversion. Stable patients, such as the patient above, should first undergo vagal maneuvers. Some common vagal maneuvers include holding your breath and bearing down (Valsalva maneuver), coughing, gagging, and immersing your face in ice-cold water. If vagal maneuvers are unsuccessful, adenosine is used both diagnostically and therapeutically. Adenosine transiently blocks the AV-node and allows the circuit to "reset." Atropine (A) is used to treat bradydysrhythmias. It inhibits vagal activity to the heart which leads to an increase in heart rate. Cardioversion (B) is not indicated in hemodynamically stable patients with PSVT. However, if unstable, the patient should undergo cardioversion. If vagal maneuvers are unsuccessful, intravenous adenosine or calcium-channel blockers, such as verapamil (D), are first-line pharmacologic options, which block the AV-node.

Which of these children with hematuria needs admission to the hospital? A 10-year-old boy with recent sore throat treated with amoxicillin, tea-colored urine, and normal urine output; UA shows microscopic hematuria and proteinuria; blood tests are normal A 12-year-old girl with lower abdominal pain, dysuria, urgency, frequency, and pink urine; UA shows microscopic hematuria, > 50 WBCs/hpf, + leukocyte esterase, + nitrites; urine pregnancy test is negative A 6-year-old boy with nausea, vomiting, bloody diarrhea, and decreased urine output; UA shows microscopic hematuria and hyaline casts; blood tests show a high WBC count, low hemoglobin, and low platelets An 8-year-old girl with diffuse mild edema, tea-colored urine, and normal urine output; UA shows microscopic hematuria and proteinuria; blood tests show a normal WBC count, hemoglobin, and platelets but low protein

Correct Answer ( C ) Explanation: This boy has hemolytic-uremic syndrome (HUS), a disorder that causes a triad of acute renal failure, thrombocytopenia, and microangiopathic hemolytic anemia. It most often results from antibiotic treatment of gastroenteritis in individuals with Escherichia coli O157:H7 infection as the dying organism releases a highly virulent, Shiga-like toxin (SLT). Symptoms of HUS are more than simple E. coli enteritis and include decreased urine output, irritability, lethargy, seizures, or encephalopathy. Unlike disseminated intravascular coagulopathy (DIC), the PT, aPTT, and fibrinogen are normal. ED management is supportive. All patients with HUS should be hospitalized. Poststreptococcal glomerulonephritis (A) results from immune complex deposits in the glomeruli after acute streptococcal infection. Asymptomatic patients with a normal blood pressure may be discharged in consultation with a pediatric nephrologist. Older children with simple cystitis (B) can be safely managed as an outpatient with oral antibiotics. Nephrotic-nephritic syndrome (D) is caused by increased permeability of the glomerular capillary wall leading to renal protein loss and its associated consequences. The hallmarks are proteinuria, hypoproteinemia, generalized edema, and hypercholesterolemia. Patients with nephrotic syndrome with only mild or moderate edema without respiratory symptoms, signs of infection, or thrombotic complications may be discharged home in consultation with a pediatric nephrologist. The treatment of nephrotic syndrome is oral corticosteroids.

A 46-year-old woman presents to her primary care provider with a complaint of severe itching on her hands and fingers. On physical examination, linear vesicles and pustules with some excoriation are seen in the web spaces of the fingers. Which of the following will confirm the suspected diagnosis? Gram stain and culture Patch testing Skin scraping and microscopy Wood's lamp and KOH prep

Correct Answer ( C ) Explanation: This patient has Scabies, which is caused by an infestation with Sarcoptes scabiei. It is usually spread by being in close contact with an infected individual. The classic symptom of scabies is generalized, severe pruritus, particularly at night. Burrows, vesicles and pustule particularly in the finger web and wrist crease are hallmark physical findings. Definitive diagnosis is made by visualization of mites, ova, or feces under microscopy. Gram stain and culture (A) should be performed on lesions suspected to have a bacterial origin; however given that this patient likely has scabies, gram stain would not provide any diagnostic value. Patch testing (B) is utilized to identify a causative agent and confirm a diagnosis of allergic contact dermatitis. Wood's lamp and KOH preps (D) are utilized to diagnose lesions caused by fungal infections of the skin.

A 22-year-old landscaper presents to the ED with a rash that began approximately three weeks ago. He reports it started with a single bump on his left forearm that has been oozing fluid. He subsequently developed lesions tracking up his arm. He denies any joint pain, decreased range of movement, fever, numbness, or weakness. On exam, he has an area of redness overlying a small papule and a rash spreading proximally as depicted in the image above. Which of the following is the most appropriate treatment for this condition? Acyclovir Hydrocortisone 1% Itraconazole Permethrin

Correct Answer ( C ) Explanation: This patient has sporotrichosis, a fungal infection transmitted by inoculation into the skin with the fungus Sporothrix schenckii. It is most commonly seen in gardeners, farmers, landscapers, and other agricultural workers. The infection is classically associated with a seemingly innocuous puncture wound from a rose bush thorn. Its characteristic rash begins with an ulcer or papule and then spreads proximally along lymphatic channels with skip lesions. Lymphocutaneous involvement, as depicted in the picture, is the most common manifestation. Treatment for this is three to six months of itraconazole. Acyclovir (A) can be used for cutaneous zoster, which looks similar to this infection. However, zoster occurs in a dermatomal distribution and presents primarily with vesicles on an erythematous base. Hydrocortisone (B) is used for various inflammatory and allergic conditions such as eczema. It is not indicated for the treatment of sporotrichosis. Permethrin (D) is commonly used in the treatment of scabies, an infection that can be identified by visualization of burrowing mites that appear as linear or s-shaped tracks in the skin, often accompanied by rows of small pimple-like insect bites. Scabies skin lesions are often found in crevices of the body such as on the webs of fingers and toes, around the genital area, and under the breasts of women.

A 32-year-old woman presents complaining of "bumps" in her vaginal area that she noticed two weeks ago. She has never had these in the past. She denies any pain. Physical exam reveals several flesh colored lesions with prominent papillae that are non-tender to palpation on the labia as shown below. Which of the following is the next best step in management? Referral to general surgery for excision Application of trichloroacetic acid Perform colposcopy with acetic acid application Perform shave biopsy for confirma

Correct Answer ( C ) Explanation: This patient is suffering from condyloma acuminatum, or genital warts, caused by an infection with the human papilloma virus (HPV), commonly types 6 and 11. Performing a colposcopy with acetic acid application will ensure that there is no infection of the cervix with condyloma acuminatum. This is important because human papilloma virus (HPV) infection is strongly associated with the development of genitourinary and rectal cancer. HPV is a sexually transmitted infection that can be prevented with vaccination. All girls and boys who are 11 or 12 years old should get the recommended series of HPV vaccine regardless of sexual history. Symptoms include flesh colored to whitish, exophytic or papillomatous growths in the genital area. In women, this can affect the vagina, cervix, vulva, oropharynx, perineum, and perianal areas. HPV can be spread via sexual contact or from mother to newborn during passage through the birth canal. Treatment includes trichloroacetic acid solution, podophyllin, cryosurgery, surgical excision, or imiquimod cream application. Application of trichloroacetic acid (A) is a viable treatment option; however, before treatment is initiated, the extent of the infection should be established. Performing a shave biopsy (C) is often not necessary as the diagnosis can be made clinically. Referral to general surgery for excision (D) may be necessary for lesions that are large and need debulking.

A 9-year-old boy presents with an inability to retract his foreskin. He has not been able to urinate for 2 days. The visible portion of the glans appears dusky. Which of the following therapies is most appropriate? Circumcision Corticosteroid cream Dorsal slit procedure Intracavernosum phenylephrine

Correct Answer ( C ) Explanation: This patient presents with phimosis and evidence of vascular compromise requiring performance of a dorsal slit procedure. Phimosis occurs when the foreskin of an uncircumcised penis becomes constricted preventing retraction of the prepuce from the glans. It results in urinary outlet obstruction, glans ischemia and infarction. Many cases are physiologic resulting from normal development. Patients typically present with an unretractable foreskin and symptoms consistent with urinary obstruction including decreased urinary stream. Management of phimosis can be difficult. Dilation of the prepuce can be performed using forceps but often does not result in relief of phimosis. When there are signs of glans vascular compromise (i.e. discoloration) a dorsal split procedure should be performed. During this procedure, the foreskin is anesthetized and incised dorsally allowing for retraction. A circumcision (A) may be necessary for definitive care but treatment should not be delayed. Patients with no signs of obstruction or vascular compromise can be treated with topical corticosteroids (B) for 6 weeks but require close follow up. Intracavernosum phenylephrine (D) is a treatment modality for priapism but plays no role in the treatment of phimosis.

Which of the following organisms is the most common cause of traveler's diarrhea? Campylobacter jejuni Enterohemorrhagic Escherichia Coli Enterotoxigenic Escherichia Coli Giardia lamblia

Correct Answer ( C ) Explanation: While viruses are responsible for the majority of infectious diarrheas in the United States. Travel to a developing country is associated with an 80% probability of bacterial diarrhea. The most common cause of traveler's diarrhea is enterotoxigenic E. Coli which is acquired by ingesting fecally contaminated food and water. Countries in Asia, Africa, Latin America and parts of the Middle East are high risk destinations. Other risk factors include travel during the rainy season and type of travel (e.g. camping, living with native inhabitants). Patients present with abrupt onset of crampy abdominal pain and watery diarrhea. Treatment consists of oral rehydration, a 3 day course of ciprofloxacin and antimotility agents, such as loperamide. Campylobacter jejuni (A) is also transmitted via ingestion of fecally-contaminated food and water. Patients may present with fever, abdominal cramping, and diarrhea that is initially watery, but may become bloody. It is most common in travelers to Southeast Asia. Enterohemorrhagic E.Coli (Ō57:H7) (B) produces Shigella-like toxins that are cytotoxic to the intestinal vascular endothelium resulting in bloody diarrhea, abdominal cramping, and low-grade fever. Antibiotics and antimotility agents should not be used in cases of suspected enterohemorrhagic E. Coli as it may increase the chance of complications (e.g. hemolytic uremic syndrome). Giardia (D) is the most common intestinal parasite in the U.S. and is associated with ingestion of contaminated water from streams or wells.

What is the most common malignancy in men age 15-35? Hodgkin's lymphoma Papillary thyroid cancer Renal cell cancer Testicular cancer

Correct Answer ( D ) Explanation: Although germ cell tumors (testicular cancer) are overall uncommon (only 1% of all male malignancies) and can occur at any age, they are the most common malignancy among men aged 15 to 35. Seminoma accounts for 40% of all germ cell tumors (GCT), whereas nonseminoma germ cell tumors (NSGCTs) account for 60%. GCT therapy serves as a model for the treatment of curable cancers and is particularly notable because GCT occurs in young men who are entering their most productive years. Because of their young age, patients who have been cured are at risk of delayed, treatment-induced toxicity. Risk factors include cryptorchidism, family history, infertility, tobacco use, and exposure to diethylstilbestrol (DES) in utero. All testicular masses should be investigated for possible cancer. Exam reveals a firm, hard, or fixed nodular mass on the testis that does not transilluminate. It may be accompanied by epididymitis and inguinal lymphadenopathy. Hodgkin's lymphoma (A) has a bimodal age distribution, young adulthood (age 15-35) and >55-years-old. It is less common then testicular cancer. Papillary thyroid cancer (B) has a female preponderance of 2.5:1, most commonly occurring during the fourth and fifth decades of life. Renal cell carcinoma (C) is approximately 50 percent more common in men compared with women. Renal cell carcinoma occurs predominantly in the sixth to eighth decade of life with median age at diagnosis around 64 years of age.

A 58-year-old man presents to your office with complaints of chronic cough and difficulty breathing. He has smoked approximately 2 packs of cigarettes per day since he was 14 years old and has not seen a doctor in many years. You order spirometry testing, which shows airflow limitation (predicted FEV1 45%). Which of the following is the most appropriate therapy? Azithromycin Guaifenesin Prednisone Tiotropium

Correct Answer ( D ) Explanation: Chronic obstructive pulmonary disease (COPD) is a progressive condition characterized by airflow limitation that causes an enhanced inflammatory response in the airways and lungs. Historically COPD was divided into three subtypes: chronic bronchitis, emphysema and asthma. Asthma is no longer grouped with COPD and current definitions of COPD do not differentiate between emphysema and chronic bronchitis. The most significant risk factor for the development of COPD is cigarette smoking. Other risk factors include secondhand smoke exposure, as well as occupational, inhalational, and environmental exposures to lung irritants. Patients generally have a lengthy smoking history and are in their fifth decade of life. They often have a chronic cough with sputum production. Diagnosis is by pulmonary function tests including spirometry. Treatment is based on the severity of the disease. Long-acting anticholinergics such as tiotropium are the mainstay of therapy. Azithromycin (A) is an antimicrobial used in conjunction with oral prednisone (C) to treat COPD exacerbations. Neither agent is used in maintenance treatment of COPD. Guaifenesin (B) is an oral expectorant that has limited clinical benefits in the treatment of COPD and may have significant adverse effects including nausea, vomiting, dizziness, headache and drowsiness.

According to JNC-8, patients older than 60 years with hypertension and no other medical history, should be treated to which of the following blood pressure goals? < 120/80 mm Hg < 130/85 mm Hg < 140/85 mm Hg < 150/90 mm Hg

Correct Answer ( D ) Explanation: Hypertension (HTN) is one of the most common diseases encountered in primary care, affecting nearly 30% of the US population. There are two main types, essential (95% of all cases) and secondary. Essential HTN begins between ages 25 and 55 years, is associated with a genetic predisposition and positive family history, and is felt to be due to repeated renal microvascular injury and an underlying hyperactive sympathetic tone. Secondary HTN is considered in those <20 or >50 years of age, or in cases of acute-onset, severe or refractory HTN. JNC-7 defines stage 1 HTN as SBP ≥ 140 mmHg and DBP ≥ 90 mmHg. In comparison, JNC-8 recommends treating hypertensive adults > 60 years old to a goal of < 150/90 mm Hg, and adults 30-59 years old to a goal of DBP < 90 mm Hg. There is insufficient evidence for a systolic recommendation in those < 60 years old and for a diastolic recommendation in those < 30 years old. As such, JNC-8 recommends for hypertensive patients < 60 years old a goal of < 140/90 mm Hg. < 120/80 mm Hg (A), < 130/85 mm Hg (B), and < 140/85 mm Hg (C) are incorrect goals in managing hypertension in patients > 60 years old.

Which of the following tests is helpful in the diagnosis for iron-deficiency anemia? Basophilic stippling Decreased serum haptoglobin Mean corpuscular volume >100 fL Serum ferritin <15 mcg/L

Correct Answer ( D ) Explanation: Iron-deficiency anemia is a common cause of chronic anemia. Symptoms can lag behind the development of the disease because changes to RBC size and hemoglobin content only occur after cytochrome and bone marrow iron stores are depleted. The majority of iron in the body is stored in hemoglobin, the reticuloendothelial system, myoglobin and bound to transferrin. Women possess less hemoglobin and myoglobin and have lower iron stores making them more susceptible to iron-deficiency anemia. There are two major etiologies of iron-deficiency: iron-store depletion and malabsorption. The prior usually involves growth in infants and adolescents, inadequate daily intake and menstruation. Malabsorption involves processes like sprue, gastrectomy and inflammatory bowel disease. Diagnosis is made by laboratory testing that is usually triggered by the presence of hypochromic, microcytic anemia. Diagnostic studies include a fasting serum iron (<50 mcg/dL), transferrin saturation percent (<10%), serum ferritin (<15 mcg/L) and total iron-binding capacity (> 400 mcg/dL) in the setting of decreased mean corpuscular volume (MCV). Depressed serum ferritin levels indicate depleted total body iron stores. These patients will also have decreased reticulocyte counts as the building blocks for new RBCs are diminished. Basophilic stippling (A) is the pathognomonic finding in lead toxicity causing sideroblastic anemia. A decreased serum haptoglobin (B) is seen in inherited hemolytic anemias (i.e. hereditary spherocytosis, glucose-6-phosphate dehydrogenase deficiency) when acute hemolytic crisis develops. A mean corpuscular volume over 100 fL (C) is indicative of megaloblastic anemia, not microcytic anemia.

A 23-year-old woman presents with concerns of tender breast enlargement. Two weeks ago, she gave birth to a healthy newborn, whom she currently breast feeds. Examination reveals general erythema, warmth and edema of the right breast. There are no superficial abnormalities, no palpable mass and no purulent nipple discharge is present. The left breast appears normal. Which of the following is the most likely diagnosis? Breast abscess Breast engorgement Inflammatory breast cancer Lactation mastitis

Correct Answer ( D ) Explanation: Mastitis generally refers to breast inflammation, which can be infectious, noninfectious or associated with inflammatory breast cancer. Of the infectious subclass, lactation mastitis is the most common and occurs within a few weeks post-delivery in postpartum women. Typical symptoms include, general body ache fever, malaise and tender breast engorgement (almost always unilateral) with erythema and induration. More serious disease may present with purulent nipple discharge. This condition may progress into or present initially as an abscess (A), a more defined and localized pocket of infection which is usually discretely palpable. Postpartum women who do not breast feed may experience symmetric breast engorgement (B) typically within 3 days post-delivery, but usually do not have the associated inflammatory symptoms as above. Although breast cancer (C) can present at many different ages, the patient's young age makes inflammatory cancer less likely. Furthermore, inflammatory breast cancer is associated with skin thickening and dimpling, as well as axillary lymphadenopathy. A blocked breast duct, inspissation, can mimic mastitis, but usually does not have an associated fever.

A previously healthy 28-year-old woman presents to your office with complaints of worsening overall muscle weakness and drooping eyelids for the past month. Physical exam reveals a mask-like face with ptosis. Sensory exam and deep tendon reflexes are normal. Which of the following is the most likely diagnosis? Bell's palsy Guillain-Barré syndrome Multiple sclerosis Myasthenia gravis

Correct Answer ( D ) Explanation: Myasthenia gravis is an autoimmune disorder of neuromuscular transmission that has two forms, ocular and generalized. Ocular myasthenia gravis presents with weakness of the eyelids and extraocular muscles. Generalized myasthenia gravis presents with a combination of ocular, bulbar, limb, and respiratory muscular symptoms. Patients generally present with ocular symptoms initially, which progress to the facial and bulbar muscles, and then to muscles in the trunk and limbs. Most cases of myasthenia gravis are idiopathic, although certain medications such as antibiotics, beta-blockers, lithium and prednisone can exacerbate symptoms. Certain findings have also been found to be associated with the disorder including predisposition to other autoimmune diseases such as rheumatoid arthritis and systemic lupus erythematosus. The approach to diagnosing myasthenia gravis is a clinical one based on history, symptoms and physical exam findings. Treatment of myasthenia gravis includes a number of modalities including use of anticholinesterase agents to treat symptoms, rapid and chronic immunomodulating treatments, and surgical treatment with thymectomy. Bell's palsy (A) is also known as idiopathic facial paralysis. Patients present with acute onset unilateral upper and lower facial paralysis. Most cases resolve slowly without intervention. Guillain-Barré syndrome (B) is a post-infectious neuropathy. Clinical manifestations include progressive, ascending, symmetric muscle weakness that begins in the lower extremities. Multiple sclerosis (C) presents with sensory and muscular symptoms that occur months or years apart and affect different locations in the body.

Which of the following statements best explains the use for penicillin in patients with acute rheumatic fever? Penicillin is only recommended for patients with carditis during their acute rheumatic fever Penicillin is recommended to prevent Group A Strep colonization, which is more common in patients with acute rheumatic fever Penicillin is used for its anti-inflammatory effect on the affected heart valves Penicillin is used to prevent future episodes of acute rheumatic fever, which can lead to worsening heart disease

Correct Answer ( D ) Explanation: Penicillin is recommended as prophylaxis for patients with rheumatic heart disease to prevent future Group A Strep infections, which can lead to recurrence of acute rheumatic fever and worsening heart disease. Prior to the availability of penicillin prophylaxis, up to 75% of patients with rheumatic heart disease had a recurrence of acute rheumatic fever. Such recurrences can cause worsening of heart disease in patients who had carditis during their initial episode of acute rheumatic fever. The risk of recurrent acute rheumatic fever decreases with time from the initial episode of acute rheumatic fever. Penicillin prophylaxis is recommended for patients with both carditis (A) and chorea during acute rheumatic fever. Although patients who presented with chorea may not have concomitant carditis, they have a 20% chance of developing carditis in the future. Colonization with Group A Strep (B) is not known to be more common in patients with acute rheumatic fever. However, decreasing Group A Strep colonization is recommended, as colonization may increase risk of recurrence of acute rheumatic fever. Penicillin is not used for its anti-inflammatory effect on the affected heart valves (C), as anti-inflammatory properties of penicillin in the absence of infection have not been described. Instead, salicylates are used to decrease inflammation.

A young woman presents to the ED with 3 days of new-onset intermittent, crampy abdominal pain. She reports one day of diarrhea and one episode of vomiting, but for the last two days, reports little to no bowel movements and increasing abdominal distension. Her medical history is significant for hyperthyroidism, irritable bowel syndrome, laparoscopically treated endometriosis, asthma and chronic tension-type headaches. Abdominal radiograph reveals excessive air in the proximal small bowel and a lack of air in the colon. Which of the following is the most likely cause of this patient's symptoms? Hyperthyroidism Irritable bowel syndrome Opioid analgesics Postoperative adhesions

Correct Answer ( D ) Explanation: Small bowel obstruction is suspected in a patient with crampy and intermittent abdominal pain, nausea, vomiting, abdominal distension, early onset diarrhea and late onset constipation. It is most commonly caused by postoperative adhesions (60%), hernia, malignancy, Crohn's disease, and radiotherapy. Fever and tachycardia may suggest an obstruction that has progressed to intestinal strangulation. The diagnosis is made by identifying dilated small loops of bowel with air fluid levels. CT scan is more sensitive and specific than plain films and can identify partial vs. complete bowel obstructions, the level and type of obstruction, and may demonstrate the cause. It is also helpful in differentiating between ileus and obstruction. Nonoperative treatment is based on the cause, but emergent care usually begins with cardiopulmonary monitoring, bowel rest, fluid resuscitation, bowel decompression (nasogastric tube), analgesics, antiemetics and surgical consultation. Intestinal strangulation is a surgical emergency and is most common in those with complete small bowel obstruction. Hyperthyroidism (A) and irritable bowel syndrome (B) are not associated causes of small bowel obstruction. Opioid analgesics (C) can lead to constipation, but, do not cause bowel obstruction.

You are counseling a mother about the potential side effects of vaccines. Which of the following is a potential side effect of the diphtheria, tetanus and acellular pertussis (DTaP) vaccine? Immune thrombocytopenia purpura Intussusception Parotid swelling Whole limb swelling

Correct Answer ( D ) Explanation: The DTaP vaccine has a 1 in 30 risk of causing entire limb swelling, redness and pain. Specifically, the pertussis component is the causative agent of limb swelling in the vaccine. The risk of this side effect increases with each subsequent booster dose. About 1 in 4 children get localized swelling, soreness, or tenderness where the shot was given, about 1 in 4 children get a fever after the vaccine and about 1 in 50 children have associated vomiting. More rare complications of the vaccine include seizures (1 in 14,000), unconsolable crying for more than 3 hours (1 in 1,000) or high fever above 105°F (1 in 16,000). Serious allergic reactions occur in less than 1 out of a million doses. Children should get the 5 doses of the DTaP vaccine at 2, 4, 6, and 15-18 months of age and at 4 years of age. Several, but not all, studies suggest that the rotavirus vaccines may cause a small increase (1 to 3 out of 100,000) in the risk of intussusception (B). The measles, mumps and rubella (MMR) vaccine has been associated with seizures (about 4 in 10,000), joint pain, temporary arthritis, parotid swelling (C), and immune thrombocytopenic purpura (A).

A 23-year-old man presents with testicular pain for 3 hours. He denies any trauma. The pain is constant, sharp and severe and is accompanied by nausea and vomiting. His vital signs are normal except for a heart rate of 110 beats/min. On examination, there is no swelling but the left testicle is extremely tender to palpation. The cremasteric reflex is absent. What management is indicated? Ceftriaxone 250 mg intramuscular X 1 and doxycycline 100 mg BID X 10 days CT scan of the abdomen and pelvis without contrast Immediate scrotal ultrasound followed by urology consultation Immediate urology consultation followed by scrotal ultrasound

Correct Answer ( D ) Explanation: This patient has signs and symptoms concerning for testicular torsion; one of the few true urologic emergencies. Torsion of the testes results from an extravaginal or intravaginal defect leading to twisting of the spermatic cord and resultant ischemia to the testicle. Although the majority of torsion occurs in the absence of trauma, MVC and straddle injuries have been associated with this disorder. Initially, twisting causes decreased venous return but eventually, arterial obstruction occurs. Physical exam often reveals a tender swollen testicle with a transverse lie that may be higher than the unaffected testicle due to shortening of the spermatic cord with twisting. An absent cremasteric reflex (elevation of the testicle caused by stroking of the inner thigh) is nearly universal (100% in patients >30 months of age). The duration of vascular obstruction affects the testicular salvage rate. Torsion recognized within 6 hours of symptom onset is associated with a salvage rate of 80-100% while delay to 24 hours or longer is almost always associated with a loss of the testicle. Because of the time sensitive nature of the disorder, urologic consultation must be immediately obtained as the only definitive way to diagnose and treat testicular torsion is with exploration and detorsion in the operating room. Manual detorsion can be attempted in the Emergency Department but is only successful 26% of the time. A scrotal ultrasound (C) can be obtained in patients with an equivocal history or physical examination but should not delay surgical consultation in patients with a high clinical suspicion. Sensitivity of ultrasound varies from 88-100% and thus, in a high-risk patient with a negative ultrasound, surgical exploration should still be considered. A CT scan of the abdomen and pelvis (B) without contrast is useful in looking for renal colic which can produce sudden, severe pain with referred pain to the testicle. However, testicular tenderness and an absent cremasteric reflex are not seen in renal colic. Ceftriaxone and doxycycline (A) are indicated in the treatment of epididymitis. Epididymitis can present with generalized testicular pain but it is usually gradual in onset and the cremasteric reflex should be intact.

A 60-year-old man presents with the ECG seen above. His blood pressure is 80/60 mm Hg. Which of the following is the most likely diagnosis? Atrial flutter Atrioventricular reentry tachycardia Supraventricular tachycardia Ventricular tachycardia

Correct Answer ( D ) Explanation: Ventricular tachycardia (VT) is present when there are >3 consecutive premature ventricular contractions occurring at a rate > 100. P waves are usually absent and the QRS complexes are wide (> 120 msec) and may be bizarre appearing. Ventricular tachycardia is classified as "monomorphic" (QRS complexes look the same) or "polymorphic" (QRS complexes have varying morphology). Sometimes it is difficult to distinguish ventricular tachycardia from supraventricular tachycardia (SVT) with aberrancy (presence of a bundle branch block). Nonetheless, any patient with a wide complex tachycardia who is hemodynamically unstable should undergo immediate electrical cardioversion. Atrioventricular reentry tachycardia (A) is a type of SVT. It is more commonly associated with regular pulse and monomorphic narrow QRS complexes. Atrial flutter (B) is more commonly associated with regular tachycardia and sawtooth appearance of P waves. It is sometimes difficult to distinguish ventricular tachycardia from supraventricular tachycardia with aberrancy. Any patient with a wide complex tachycardia who is hemodynamically unstable should be treated as having ventricular tachycardia. Some electrocardiographic characteristics that support VT over SVT (C) are extreme left axis deviation, QRS concordance (all QRS complexes from V1 to V6 are either all positive or all negative), and fusion or capture beats, which indicates AV dissociation.

A 64-year-old man presents with a cough and shortness of breath for 2 weeks. His chest X-ray is shown above. Which of the following tests would indicate an infectious process as the cause of the above finding? Pleural fluid to serum LDH ratio < 0.6 Pleural fluid to serum protein ratio < 0.1 Pleural fluid to serum protein ratio > 0.5 Pleural fluid WBC 5,000 cells/mm3

Explanation: The patient presents with a large pleural effusion that may be secondary to a pneumonia and a pleural fluid:serum protein ratio > 0.5 indicates the presence of an exudative effusion consistent with an infectious cause. Pleural effusions can result from a number of different pathologies. This includes CHF, malignancy, bacterial pneumonia, pulmonary embolism and pancreatitis. In developing countries, tuberculosis infection is the most common cause of pleural effusions. Pleural effusions can either be transudative (CHF, cirrhosis, pulmonary embolism) or exudative (malignancy, pneumonia, pulmonary embolism, pancreatitis). Patients will often present with dyspnea and pleuritic chest pain and dullness to percussion over the effusion. It is often difficult clinically to determine the cause of the effusion but testing of a sample of the fluid can differentiate exudative from transudative processes. Light's criteria (see table) is typically used for this purpose. If a single criterion is consistent with an exudative process, the fluid is exudative in origin. A pleural fluid:serum LDH < 0.6 (A) and a pleural fluid:serum protein < 0.1 (B) are both consistent with transudative processes. A pleural fluid white blood cell count greater than 10,000 cells/mm3 is indicative of an exudative effusion not a pleural fluid WBC 5,000 cells/mm3 (D).


Related study sets

Reseach2- Quantitative and Qualitative Research

View Set

tcp/ip illustrated volume 1: chapter 5

View Set

Pretest: Solving Quadratic Equations

View Set

Exam 1 Prep Chapters 2-7 Finance

View Set